Download as pdf or txt
Download as pdf or txt
You are on page 1of 135

CHEMISTRY

GRADE X

ACADEMIC WINDOW
Year 2020- 2021

NAME ----------------------------------------

Page 1 of 135
A NOTE FROM THE HOD

Chemistry has been called the science of what things are. Its intent is the exploration of the nature of
the materials that fabricate our physical environment, why they hold the different properties that depict
them, how their atomic structure may be fathomed, how they may be manipulated and changed.
Chemistry is the main incentive for the life that we live today; it has provided modern society with
facilities that make our day to day lives easier.

Some students seem naturally enthusiastic about learning, but many needs or expect their instructors
to inspire, challenge, and stimulate them. There is no single magical formula for motivating students.
Students learn by doing, making, writing, designing, creating, solving. So, we should ensure
opportunities by assigning tasks that are neither too easy nor too difficult.

This academic window is planned as a skill-based questionnaire that emphasis on thinking skills,
experimental skills and application skills and it consist of the following units– Index, Exam wise
syllabus, Glossary, Chapter names and sample papers for Formative and summative assessments.
Glossary explains all the short forms and symbols involved in each chapter, it strengthens the
students understanding the type of questions. Synopsis given in the beginning of each chapter
includes summary of the relevant chapter. The explanation of concepts is supported by photographs,
diagrams and tables. To strengthen this, HOTS questions, Multi-disciplinary and Value based
questions are included in it. The explanation of concepts is supported by photographs, diagrams and
tables. Based on the updated Question pattern by CBSE, we have included MCQ and Assertion-
Reason questions in each chapter.

I take immense pleasure in bringing out this academic window which can play an instrumental role in
developing the skills in chemistry.

Regards,

Ms Anita Roni Thomas

HOD, Department of Chemistry

DPS Sharjah

Page 2 of 135
GLOSSARY

FACTUAL / SIMPLE C

ABOVE AVERAGE U

INTERESTING! A

HOT A

MULTIDISCIPLINARY C/U

Page 3 of 135
INDEX

S.No CONTENT PAGE

1 5
CURRICULUM

2. CHEMICAL REACTIONS AND EQUATIONS 7

3. ACIDS, BASES AND SALTS 25

4. METALS AND NON-METALS 46

5. CARBON AND ITS COMPOUNDS 62

6. PERIODIC CLASSIFICATION OF ELEMENTS 79

7. PRACTICAL EXPERIMENTS 90

8. SAMPLE QUESTION PAPERS 121

Page 4 of 135
GRADE X- CHEMISTRY CURRICULUM

Unit I: Chemical Substances – Nature and Behaviour

Chemical reactions:
Chemical equation, Balanced chemical equation, implications of a balanced chemical
equation, types of chemical reactions: combination, decomposition, displacement,
double displacement, precipitation, neutralization, oxidation and reduction.
Acids, bases and salts:
Their definitions in terms of furnishing of H+ and OH– ions,
General properties, examples and uses, concept of pH scale (Definition relating to
logarithm not required), importance of pH in everyday life; preparation and uses of
Sodium Hydroxide, bleaching powder, Baking soda, Washing soda and Plaster of
Paris.
Metals and nonmetals:
Properties of metals and non-metals; Reactivity series;
Formation and properties of ionic compounds; Basic metallurgical processes;
Corrosion and its prevention.
Carbon compounds:
Covalent bonding in carbon compounds. Versatile nature of carbon. Homologous
series. Nomenclature of carbon compounds containing functional groups (halogens,
alcohol, ketones, aldehydes, alkanes and alkynes), difference
between saturated hydrocarbons and unsaturated hydrocarbons. Chemical properties of
carbon compounds (combustion, oxidation, addition and substitution reaction).
Ethanol and Ethanoic acid (only properties and uses), soaps and detergents.
Periodic classification of elements:
Need for classification, Early attempts at classification of elements (Dobereiner’s
Triads, Newland’s Law of Octaves, Mendeleev’s Periodic Table), Modern
periodic table, gradation in properties, valency,
atomic number, metallic and non-metallic properties.

LIST OF EXPERIMENTS
1. . Finding the pH of the following samples by using pH paper/universal indicator:
(i) Dilute Hydrochloric Acid
(ii) Dilute NaOH solution
(iii) Dilute Ethanoic Acid solution
(iv) Lemon juice

Page 5 of 135
(v) Water
(vi) Dilute Hydrogen Carbonate solution
2 . Studying the properties of acids and bases (HCl & NaOH) on the basis of their
reaction with:
a) Litmus solution (Blue/Red)
b) Zinc metal
c) Solid sodium carbonate
3. Performing and observing the following reactions and classifying them into:
A. Combination reaction
B. Decomposition reaction
C. Displacement reaction
D. Double displacement reaction
(i) Action of water on quick lime
(ii) Action of heat on ferrous sulphate crystals
(iii) Iron nails kept in copper sulphate solution
(iv) Reaction between sodium sulphate and barium chloride solutions
4 . Observing the action of Zn, Fe, Cu and Al metals on the following salt solutions:
i) ZnSO4 (aq)
ii) FeSO4 (aq)
iii) CuSO4 (aq)
iv) Al2 (SO4)3 (aq)
Arranging Zn, Fe, Cu and Al (metals) in the decreasing order of reactivity based on the
above
result.
5 . Study of the following properties of acetic acid (ethanoic acid):
i) odour
ii) solubility in water
iii) effect on litmus
iv) reaction with Sodium Hydrogen Carbonate
6. Study of the comparative cleaning capacity of a sample of soap in soft and hard
water

Page 6 of 135
CHAPTER 1
Chemical reactions and equations

Synopsis:

Chemical Reactions
Transformation of a substance into a new substance or many new substances is called a chemical
reaction. For example, water (which is the compound formed when Hydrogen and Oxygen bind
together), the chemical reaction can be written as:
2H2 + O2 2 H2O

Chemical Equation
It is the method of representing a chemical reaction with the help of symbols and formulae of
substances involved.

The substances which react are called reactants and the new substance which forms is
called the product. For example,

In a chemical equation, the number of atoms on the reactant side should be the same as the
number of atoms on the product side. The reaction should be balanced.

Example : Ag2O Ag + O2 (unbalanced)


2Ag2O 4Ag + O2 (balanced)

Combination reaction: A combination reaction occurs when simpler substances combine to


form a single substance.

2H2 + O 2 2H2O

Page 7 of 135
Decomposition reaction: A decomposition reaction occurs when compounds are split into simpler
substances.

2H2O 2H2 + O2
electricity

Decomposition is the reverse of combination.

Displacement reactions: A more reactive metal (higher in the reactivity series) will displace a less
reactive metal from its salt solution.

CuSO4 (aq) + Zn (s) ZnSO4 (aq) + Cu (s)

Blue copper sulphate solution reacts with zinc to give colourless zinc sulphate solution and solid
copper. Thus Zn displaces Cu in the salt form.

Double displacement reactions: In a double displacement reaction, two substances react by


exchanging their radicals.

In a precipitation reaction, two compounds in their aqueous state react to form an insoluble residue
(precipitate) as one of the reaction product.

NaCl + AgNO3 + NaNO3 + AgCl

Page 8 of 135
Oxidation and reduction (Redox)

Oxidation is the gain of oxygen/loss of hydrogen. Reduction is the loss of oxygen/gain of hydrogen.

Reduction and oxidation always occur together, and these reactions are often called redox reactions.

Exothermic and endothermic reaction

Exothermic reactions give out heat.

Endothermic reactions take in heat.

Page 9 of 135
Corrosion: When the surface of a metal is attacked by air, water, or any other substance around it, it
is said to corrode and the phenomenon is known as corrosion. Rusting involves the reaction of iron
with oxygen and water to form hydrated iron (III) oxide which is rust.Iron becomes brown on
corrosion.
4Fe + x .H2O + 3O2 → 2Fe2O3.x H2O

Rusting can be prevented by oiling (greasing), painting, Galvanizing (coating with zinc), plating (e.g.
with chromium)

Copper becomes green due to formation of basic copper carbonate


2Cu + H2O + CO2 + O2 → CuCO3.Cu(OH)2

Rancidity: Food materials containing oil or fat get oxidized on exposure to air and their smell and the
taste changes. This is called rancidity.

Rancidity can be prevented by: storing the food in airtight container, adding an antioxidant or
flushing the food item with an inert gas like nitrogen.

Page 10 of 135
Other examples of anti-oxidants: BHA (Butylated hydroxyl anisole), BHT (Butylated hydroxyl
toluene)

Page 11 of 135
ACTIVITY 1.1

Aim: To show burning of magnesium ribbon in air is a combination reaction.

Procedure: Take a clean strip of magnesium ribbon and introduce it in the flame of a burner with the
help of tongs.

Observation:

i)

ii)

Reaction involved:

Conclusion:

ACTIVITY 1.2

Aim: To show the reaction between lead nitrate and potassium iodide is a double displacement
reaction (precipitation reaction).
Procedure:

• Take lead nitrate solution in a test tube.

• Add potassium iodide solution to it

Observation:

Reaction involved:

Conclusion:

ACTIVITY 1.3

Aim: To show the displacement reaction between zinc granules and dil.HCl

Procedure:
• Take 4-5 zinc granules in a conical flask.
• Add dil.HCl over it.
• Touch the conical flask and observe the change in temperature
Page 12 of 135
Observation:

Reaction involved :

Conclusion:

Page 13 of 135
ACTIVITY 1.4

Aim: To show the combination reaction between calcium oxide and water is an exothermic reaction.

Procedure:
• Take 5 g of Quick lime (calcium oxide) in a beaker.
• Add water to it slowly.
• Touch the beaker and note the change in temperature.

Observation:

Reaction involved:

Conclusion:

ACTIVITY 1.5

Aim: To show decomposition reaction of ferrous sulphate.

Procedure:

• Take 2 g of ferrous sulphate crystals in a dry test tube.


• Heat the crystals over the flame of a burner for some time
• Observe the crystals after heating.

Observation:

i)

ii)

Reaction involved:

Conclusion:
i)
ii)

Page 14 of 135
ACTIVITY 1.6

Aim: demonstrate electrolysis of water.

Procedure:
• Take a plastic mug and drill 2 holes at its base.
• Fit rubber stoppers into the holes and insert 2 carbon electrodes.
• Connect it to a 6V battery.
• Fill the mug with water such that the electrodes are completely immersed.
• Add a few drops of dil.H2SO4 to water (to increase its conductivity)
• Invert 2 test tubes filled with water over the electrodes.
• Switch on the current and leave the apparatus undisturbed

Observation:

i)

ii)

Reaction
involved:

Conclusion:

i)

ii)

TEST FOR HYDROGEN:

TEST FOR OXYGEN:

DIAGRAM:

Page 15 of 135
ACTIVITY 1.7

Aim: To show photochemical decomposition of silver chloride.

Procedure:

• Take 2 g of silver chloride in a china dish.


• Place it in sunlight for a few hours.

Observation:

Reaction:

Conclusion:

Page 16 of 135
ACTIVITY 1.8

Aim: To show the oxidation reaction in copper.

Procedure:
• Take 2 g of copper powder in a china dish.
• Place it on a china dish and heat it over a burner.

Observation:

Reaction involved:

Conclusion:

Page 17 of 135
EXERCISE

A MULTIPLE CHOICE QUESTIONS


. 1 mark each.

1 A reaction in which exchange of radicals takes place


a) Decomposition reaction
b) Single displacement reaction
c) Double displacement reaction
d)Combination reaction
2 What happens when dilute hydrochloric acid is added to iron filings?
a) Hydrogen gas and iron chloride are produced
b) Chlorine gas and iron hydroxide are produced
c) No reaction takes place
d) Iron salt and water are produced
3 Which of the statement about the reaction below are incorrect?
2 PbO + C → 2Pb + CO2
i)Lead is getting reduced
ii)CO2 is getting oxidized
iii)Carbon is getting oxidized
iv)Lead Oxide is getting reduced
a) i and ii
b) i and iii
c) i, ii and iii
d) All of the above
4 Which one of the following words best describes the energy change when a substance is burned?
a) Endothermic
b) Neutralization
c) Exothermic
d) Reduction

5 A student studied the reaction of citric acid with sodium hydrogen carbonate. She put a solution of citric acid in
a plastic cup and measured its temperature. Then she added sodium hydrogen carbonate powder and measured
the temperature again. The temperature of the reaction mixture decreased. Which one of these statements about
this reaction is correct?
a) No change in temperature
b) The reaction is exothermic
c) The reaction is endothermic
d) None of these
B ASSERTION & REASON
1 mark each.

In the following questions, the assertion and reason have been put forward. Read the statements
Page 18 of 135
carefully and choose the correct alternative from the following.
a. Both the assertion and reason are true, and Reason is the correct explanation of assertion.
b. Both the assertion and reason are true, but Reason is not the correct explanation of assertion.
c. Assertion is true, but reason is false.
d. Both assertion and Reason are false
1 Assertion: AgBr is used on photographic and x-ray films.
Reason : AgBr is photosensitive and changes to Ag and bromine in the presence of sunlight and undergoes
decomposition reaction.

2 Assertion: Magnesium ribbon keeps on burning in atmosphere of nitrogen.


Reason : Magnesium reacts with nitrogen to form magnesium nitride and this reaction is combination
reaction.

3 Assertion: Zinc reacts with sulphuric acid to form zinc sulphate and hydrogen gas and it is a displacement
reaction.
Reason : Zinc reacts with oxygen to form zinc oxide.

4 Assertion: Lead nitrate on thermal decomposition gives lead oxide, brown coloured nitrogen dioxide and
oxygen gas.
Reason : : Lead nitrate reacts with KI to form an yellow precipitate of PbI and the reaction is a double
displacement reaction.

5 Assertion: The colour of ferrous sulphate crystals before heating is green and after heating is reddish brown.
Reason : Ferrous sulphate undergoes thermal decomposition reaction.

C
VERY SHORT ANSWER TYPE QUESTIONS (VSA) 1 mark each.

1 Hydrogen being a highly inflammable gas and oxygen being a supporter of combustion, yet water
which is a compound made up of oxygen and hydrogen is used to extinguish fire. Why?

2 State the law that governs balancing of chemical equations.

Page 19 of 135
3 Balance the following equations:
a) Mg + N2 → Mg3N2

b) ZnCO3 + HNO3 → Zn(NO3)2 + H2O + CO2


4 Consider the following reactions:
CuSO4(aq) + Fe(s ) → FeSO4(aq) + Cu(s)
FeSO4(aq) + Zn(s) → ZnSO4(aq) + Fe(s)

Based on the above reactions indicate the most reactive and the least reactive metal out of zinc,
copper and iron.

5 What change in color is observed when white silver chloride is left exposed to sunlight ? State the
type of chemical reaction in this change.

D SHORT ANSWER TYPE QUESTIONS (SA) 2 marks each.


.

1 Write any two observations in an activity which may suggest that a chemical reaction has taken
place.? Give an example to support your answer.

2 An aluminium can is used to store ferrous sulphate solution. It is observed that in a few days holes
appear in the can. Explain the observation and write chemical equation to support your answer.

Page 20 of 135
3 A white salt on heating decomposes to give brown fumes and a residue is left behind.
a) Name the salt.
b) Write the equation for the decomposition reaction.

4 Identify the oxidized and reduced species in the following reactions

a) H2S + Cl2 → S + 2HCl

b) MnO2 + 4HCl MnCl2 + 2H2O + Cl2


5 John was reading the ingredients on a packet of fried crisps which contained an antioxidant. He was
wondering what is an antioxidant and what is it used for? Can you help him understand the importance of
adding antioxidants?

E. SHORT ANSWER QUESTIONS (SA) 3 marks each.

1 Water is added to a small amount of quick lime in a beaker.


a. Write two changes that are observed.
b. Write the chemical equation
c. Name the type of the reaction.

2 Write the chemical equation for each of the following reactions and balance them:
a) Phosphorous burns in chlorine gas to form phosphorous pentachloride

b) Aqueous solutions of sulphuric acid and sodium hydroxide react to form aqueous sodium sulphate
and water.

c) Calcium carbonate on heating decomposes to form calcium oxide and carbon dioxide

3 A student writes the balanced equation for the reaction of magnesium with nitrogen as follows:

3Mg + 2N → Mg3N2
What is wrong in this equation? Write the correct balanced equation.

4 A solution of compound ‘X’ in water was added to the solution of sodium chloride. It formed a white
precipitate of the compound ‘Y’. It was filtered and kept in a china dish in sunlight for drying but it turned
grey. Explain the possible reason for this. Identify the compounds X and Y and write the reactions involved.

Page 21 of 135
5 2g of ferrous sulphate crystals are heated in a dry boiling tube.
a) List any two observations

b) Name the type of chemical reactions taking place.

c) Write the chemical equation for the reaction.

Page 22 of 135
F. LONG ANSWER QUESTIONS (LA) 5 marks each.

1 a) Corrosion is a process observed on most of the metals which causes damage and loss to metallic
properties. Explain what is meant by corrosion?
b) What is the name given to the corrosion of iron?
c) What is the colour of coating the corrosive substance formed on silver and copper?
d) What damage is caused by corrosion?
e) How can we prevent corrosion?

2 A metal nitrate A on heating gives yellowish brown colored metal oxide along with a brown gas B and a
colorless gas C. Aqueous solution of A on reaction with potassium iodide forms a yellow precipitate of
the compound D. Identify A, B, C ,and D. Also identify the types of both the reactions.

3 When a black metal compound XO is heated with a colorless gas Y2, then metal X and compound Y2O
are formed. Metal X is red- brown in color which does not react with dilute acids at all. Gas Y2 can be
prepared by the action of a dilute acid on any active metal. The compound Y2O is a liquid at room
temperature which can turn anhydrous copper sulphate blue.
a) Name the metal X, gas Y2, compound XO and Y2O.
b) Write the chemical equation of the reaction which takes place on heating XO with Y2.

Page 23 of 135
Achieved Working Needs
towards reinforcement

I can recognize chemical changes.


I can write balanced chemical
equations to represent a chemical
reaction.
I can classify chemical reactions and
list examples of different types of
chemical reactions.
I can relate my learning to real life
when I look at metal articles which
corrode and food items which
become rancid.
Teacher’s feedback:

Student’s feedback:

Next step in Learning:

WORK WELL DONE WORK ADEQUATELY WORK REQUIRES MORE


ATTEMPTED
PRACTICE AND EFFORT

REMARKS :

Signature: Date:

Page 24 of 135
CHAPTER 2
Acids, Bases and Salts

S.NO. INDICATORS ACIDS BASES


1 Red litmus solution Remains red Turns blue
2 Blue litmus solution Turns red Remains blue
3 Phenolphthalein solution Colourless Pink
4 Methyl orange solution Pink/Red Yellow

Indicators – There are 2 types of indicators:


Visual Indicator - –Indicates whether a substance is acidic or basic in nature by change in color.
Ex: Turmeric, a natural indicator, turns reddish brown in basic medium but remains yellow in acidic
and neutral medium.

Olfactory Indicator - –Odour changes in acidic or basic media. For e.g. –clove oil, vanilla essence.

Page 25 of 135
REACTION WITH METALS
Acid + metal → salt + hydrogen

Eg. Zn +2HCl →ZnCl2 +H2

Base + metal → salt + hydrogen


(only for Na, K)-With other metals it does not evolve hydrogen gas.

Eg. Zn +2NaOH (heat) → Na2ZnO2+H2

Reaction with metal carbonate–

Acid + metal carbonate / metal hydrogen carbonate → salt + carbon dioxide + water

Eg. Na2CO3 + 2HCl → NaCl + H2O +CO2

NaHCO3 + HCl → NaCl + H2O +CO2

Page 26 of 135
Neutralization reaction –
Acid + base → salt + water

Eg. HCl + NaOH → NaCl + H2O

Reaction with metal oxides / nonmetal oxides

Acid + metal oxide → salt + water

CuO + 2HCl → Cu Cl2 + H2O

Base + nonmetallic oxide  salt and water

Ca (OH)2 + CO2 → CaCO3 + H2O

Acids and bases in water -


–Acid produces H+ ions, H+ ions cannot exist alone. They combine with water molecules to form
H3O+.

Acid +water → decrease in H3O+ion concentration per unit volume → dilute acid.

Base + water → decrease in OH- ion concentration per unit volume → dilute base.

Note – Process of dissolving acid/base in water is highly exothermic. Hence, acid/base must be
added dropwise into water with constant stirring so that the heat generated spreads over in water. If
water is added to conc acid then the heat generated may cause the mixture to splash out and cause
burns.

pH scale – Acids show pH value less than 7. Bases show pH value greater than 7.

Page 27 of 135
Importance of pH-

1. Our body works within pH range of 7.0 to 7.8.


2. Acid rain with pH 5.6, affects animals.
3. Plants need specific range of pH of soil.
4. Acids produced by stomach helps in digestion. If too much acid is formed, we take antacids.
Eg.milk of magnesia.
5. Tooth decay starts when pH in mouth is lower than 5.5. Bacteria present in mouth produce
acids by degradation of sugar and food particles remaining in the mouth. Rinse your mouth
after eating. Use toothpastes, which are basic.
6. Bee-stings acids which causes pain and irritation. Use of baking soda on the sting area gives
relief. Stinging hair of nettle leaves inject methanoic acid causing burning pain.

SALTS

Salts belonging to the same positive or negative radicals are said to belong to a family.
Eg.NaCl , Na2SO4 - –belong to sodium family.
NaCl ,KCl - Belong to chlorine family.
pH of salts –
a) salts of a strong acid and strong base are neutral with pH =7
b) salts of a strong acid and weak base are acidic with pH < 7
c) salts of a strong base and weak acid are neutral with pH > 7

Page 28 of 135
Common salt →NaCl is found in sea. It is an important raw material for making NaOH, baking
soda, washing soda, bleaching powder etc.

Sodium hydroxide - – NaOH

Preparation : Chlor – Alkali Process.


Aqueous Solution for Sodium Chloride (Brine) is electrolyzed. Products are chlorine and an alkali .
(Electrolysis)
2 NaCl (aq) + 2H2O(l) → 2 NaOH (aq) + Cl2 (g) + H2(g)

Beaching Powder : CaOCl2 , Calcium oxy Chloride

Preparation: Action of Cl2 on dry slaked lime Ca (OH)2

313K
Ca(OH)2 + Cl2 ------------ → CaO Cl2 + H2O
Uses:

• For bleaching cotton and linen in the textile industry, bleaching wood pulp in paper
industry, bleaching washed clothes in laundry.

• As an oxidizing agent in Chemical industries

• For disinfecting drinking water

Baking Soda– NaHCO3 – Sodium hydrogen Carbonate

Page 29 of 135
Preparation: Raw Materials: NaCl, NH3, H2O, CO2
Equation: NaCl + H2O + CO2 + NH3 -> NH4Cl + NaHCO3 (less soluble separator out from the
solution)
Properties: Non-corrosive base
On heating gives out CO2 and Na2 CO3 and Na2 CO3 is found (bitter in taste)
Heat
2NaHCO3 → Na2CO3 + H2O + CO2

Uses:

• Used as an antacid, being alkaline neutralizes excess acid in the stomach

• Used in Soda-acid fire extinguisher

• For making baking powder (mixture of baking Soda + tartaric acid). When baking powder
is mixed with water or heated following reaction takes place

• NaHCO3 + H+ → CO2 + H2O + Sodium Salt of acid (from acid)

• CO2 produced in the reaction makes the cake fluffy, soft and spongy

Washing Soda (Na2 CO3.10 H2O) – Sodium carbonate decahydrate

Preparation: On heating baking soda, we get Sodium Carbonate. It is crystallized on adding water.

Heat
2NaHCO3 → Na2CO3 + H2O + CO2

Na2 CO3 + 10 H2O → Na2CO3.10 H2O


(10H2O is water of crystallization)

Page 30 of 135
Uses:

• Used in making glass, soap an paper

• Used in manufacture of Sodium Compounds such as boran

• As Cleansing agent for domestic purposes

• To remove permanent hardness of water

Water of Crystallization: is the fixed number of water molecules present in one formulae unit of a
salt.
e.g. CuSO4. 5 H2O – Copper Sulphate
Na2CO3. 10H2O – Washing Soda
CaSO4. 2H2O – Gypsum

Decrystallise
E.g., CuSO4. 5H2O → CuSO4 + 5H2O
Blue color on heating white

Re-crystalize
CuSO4 + 5H2O → CuSO4. 5H2O
White Blue
Plaster of Paris: CaSO4 ½ H2O (Calcium Sulphate Hemihydrate)
373 K
CaSO4. 2H2O → CaSO4 ½ H2O + 1 ½ H2O gypsum

Uses: Used in plastering fractured bone

Reason: Plaster of Paris on mixing with water changes to gypsum and sets into hard solid mass.
* ½ H2O- as water of Crystallization is shown.
Explanation: 2CaSO4 molecules are attached with one water molecule hence 2 units share one
molecule of water or one unit share half a molecule of water.

Used in making toys, decoration materials

Page 31 of 135
ACTIVITY 2.1

Aim: To show that onion is an olfactory & visual indicator.

Procedure:
• Take the juice of crushed onions in 2 watch-glasses.
• Add a few drops of dil.HCl to one of the watch-glasses.
• Add a few drops of dil.NaOH to the other watch glass.
• Note the change in smell/colour in the two watch-glasses.

Observation:

i)

ii)

Conclusion:

ACTIVITY 2.2

Aim: To show that acids react with metals to liberate hydrogen gas.

Procedure:
• Take 2 g of zinc granules in a test-tube and fix a 2-holes rubber cork I its mouth.
• Through one hole pass a thistle funnel and through the other a doubly bent delivery tube.
• Immerse the other end of the delivery tube in a trough containing soap water.
• Add dil.HCl though the thistle funnel.
• Bring a burning candle near the soap bubbles.

Observation:

i)

ii)

iii)

Reaction involved:

Page 32 of 135
Conclusion:

ACTIVITY 2.3

Aim: To show the reaction of acids with metal carbonates/bicarbonates.

Procedure:
• Take 2 test-tubes A and B containing dil.HCl.
• Add sodium carbonate to test tube A and sodium bicarbonate to test tube B.
• Pass the gas evolved through a test tube C containing lime water.

Observation in test tube A and B:

i)

ii)

Reaction involved:

Conclusion:

Observation in test tube C:

i)

ii)

iii)

Reaction involved:

Conclusion:

ACTIVITY 2.4
Aim: To show the neutralization reaction between acid and base.
Procedure:
• Take 2 ml of dilute NaOH in a test tube and add a few drops of phenolphthalein it.
• Now add a few drops of dil.HCl to the above solution .
• Again add a few drops of dilute NaOH to the above solution.

Page 33 of 135
Observation:
i)

ii)

iii)
Reaction involved:
Conclusion:

ACTIVITY 2.5

Aim: To study the neutralization reaction of copper oxide with dil.HCl.


Procedure:
• Take small amount of copper oxide in a conical flask.
• Add dil.HCl to it slowly.
• Observe the change in colour.

Observation:

Reaction involved:
Conclusion:

ACTIVITY 2.6

Aim: To show that solutions of acids/bases conduct electricity.

Procedure:
• Take 2 iron nails and fix them on 2 rubber corks and place them in a beaker.
• Connect the nails to a 6V battery through a switch and a bulb.
• Pour dil.HCl into the beaker and switch on the current.
• Repeat the activity separately for dil.NaOH, alcohol solution and glucose solution.

Observation:

i)

ii)

Conclusion:
Page 34 of 135
ACTIVITY 2.7

Aim: To show that HCl shows acidic property only in aqueous solutions.
Procedure:
• Take 10 g of NaCl in a dry test tube and fix a 2-holed cork in its mouth.
• Through one hole insert a thistle funnel and through the other a delivery tube.
• Add conc.H2SO4 slowly through the thistle funnel.
• Test the gas evolved by bringing a dry and moist blue litmus paper near the mouth of the
delivery tube.

Observation:

i)

ii)

Reaction involved:
Conclusion:

i)

ii)

ACTIVITY 2.8

Aim: To show that dissolution of an acid/base in water is an exothermic process.

Procedure:
• Take 2 beakers marked A and B , containing 50 ml of water.
• Add a few drops of conc.H2SO4 in beaker A and a few NaOH pellets in beaker B.
• Swirl the beakers and touch the bottoms of the beakers.

Observation:

Conclusion:

ACTIVITY 2.9

Aim: To show that crystalline salts contain water of crystallization.

Procedure:

Page 35 of 135
• Take a few crystals of copper sulphate in a dry boiling test tube and observe its colour.
• Heat the crystals by bringing the test tube over the flame.
• Cool the crystals and add a few drops of water to it.

Observation:

i)

ii)

iii)

Reaction involved:

Conclusion:

Page 36 of 135
EXERCISE

A MULTIPLE CHOICE QUESTIONS (1Mark each)

. What happens when an acid reacts with metal oxide?


1 A. Salt and water is formed
B. Metal hydride is formed
C. Oxyacid will be formed
D. Salt and Hydrogen gas is formed

. Which of the following is the strongest acid in the world?


2 A. Hydrochloric acid
B. Nitric acid
C. Sulphuric acid
D. Carbonic acid

3 Which statement is correct regarding acids?


l. Acid molecule always has Hydrogen in its formula.
ll. Acid increases the concentration of hydrogen atoms or hydronium atoms in water.
Ill. Acids have a pH value of less than 7.
IV. The acid in the stomach helps in the digestion of food.
A. Only I
B. Both Il and IV
C. l, Ill and IV
D. All the above

4 Which of the following is a battery acid?


A. Sulphuric acid diluted with water.
B. Carboxylic acid mixed with water.
C. Concentrated hydrochloric acid.
D. Concentrated sulphuric acid.

5 What are Olfactory Indicators?


A. Substances whose colour changes in acidic or basic media.
B. A pH indicator made of a solution of several compounds.
C. A man-made chemical substance that can be used as acid-base indicators.
D. Substances whose odour changes in acidic or basic media.

B ASSERTION & REASON (1Mark each)

In the following questions, the assertion and reason have been put forward. Read the statements
carefully and choose the correct alternative from the following.
a. Both the assertion and reason are true, and Reason is the correct explanation of assertion.
Page 37 of 135
b. Both the assertion and reason are true, but Reason is not the correct explanation of assertion.
c. Assertion is true, but reason is false.
d. Both assertion and Reason are false.

1 Assertion (A): On passing excess of CO2 through lime water it turns first milky and then
colourless
Reason (R): It is due to the formation of insoluble CaCO3 and then soluble Ca(HCO3)2.

2 Assertion (A): pH solution is also called universal indicator


Reason (R): It ranges from 0-14.

3 Assertion (A): Acids react with most metals to liberate hydrogen gas.
Reason (R): They liberate hydrogen gas when they are less reactive than hydrogen.

4 Assertion (A): Acetic acid, sulphuric acid and citric acids are strong acids.
Reason (R): As all three compounds dissolve in water to give hydronium ions.

5 Assertion (A): A reaction between Hydrochloric acid with caustic soda is a neutralization
reaction .
Reason (R): As salt and water are the two products formed.

C VERY SHORT ANSWER TYPE QUESTIONS (1 Mark each)

1. Dry ammonia gas has no action on litmus paper, but a solution of ammonia in water turns red
litmus paper to blue. Why is it so?

Page 38 of 135
2. What is the color change observed when:
a) Blue litmus is dipped in acetic acid
b) A few drops of phenolphthalein are added to calcium hydroxide

c) Methyl orange is added to sulphuric acid


d) Red litmus is dipped in a solution of KOH
3 A substance X when reacted with an acid produced a gas that turned limewater milky. Identify
X and the gas evolved

4 How will you test the gas which is liberated when hydrochloric acid react with an active metal?

5 Give the chemical equation for the reaction when zinc metal is added to NaOH

D. SHORT ANSWER QUESTIONS (SA) 2 marks each.

1 What is meant by water of crystallization? Name three substances having the same.

2 What is plaster of Paris? What use does it find in the medical field?

Page 39 of 135
3. 15ml of water and 10 ml of sulphuric acid are to be mixed in a beaker.
a) State the method that should be followed with reason.
b) What is this process called.

4 The soil in a field is highly acidic. List any two materials which can be added to this soil to
reduce its acidity. Give reason for your choice.

5 A magician has three glasses A, B and C, all containing colourless liquids. On transferring the
contents of A to B the solution turns pink. This pink solution when added to C turns colourless
again. Identify A, B and C.

6 The oxide of a metal M was water soluble. When a blue litmus strip was dipped in this solution,
it did not show any change in colour. Predict the nature of oxide and explain.

7 `A` is a soluble acidic oxide and `B` is a soluble base. Compared to pH of pure water. What will
be the pH of (a) solution of A (b) solution of B?

E. LONG ANSWER QUESTIONS (LA) 3 marks each.

1 Explain the action of dilute hydrochloric acid on the following with chemical equations.

a) Magnesium ribbon b) Sodium hydroxide c) Crushed egg shells.

2 Explain why sulphuric acid conducts electricity whereas alcohol does not.

Page 40 of 135
3 What happens when an acid or base is dissolved in water? Explain why acid must be added to
water and not vice versa.

4 How is bleaching powder manufactured? Give two uses of each of the following and give their
chemical names and formulae.

a. Bleaching powder
b. Baking soda
c. Washing soda
5. Why is Plaster of Paris written as CaSO4 • ½ H2O? How is it possible to have half a water
molecule attached to CaSO4?

6 A compound `X` on electrolysis in aqueous solution produces a strong base. `Y` along with two
gases `A` and `B`. `B` is used in manufacture of bleaching powder. Identify X, Y, A and B.
Write chemical equations.

7 A yellow powder X gives a pungent smell if left open in air. It is prepared by the reaction of dry
compound Y with chlorine gas. It is used for disinfecting drinking water. Identify X and Y. and
write the reaction involved.

8 When CO2 gas pass through saturated solution of ammonical brine, two compounds `X` and
`Y` are formed. `Y` is used as antacid and decomposes to form another solid `Z`. Identify `X`,
`Y`, `Z`
and write chemical equations.

F. VERY LONG ANSWER QUESTIONS (VLA) 5 marks each.

1 a ) State the chemical properties on which the following uses of baking soda are based.

i) as an antacid

ii) as a soda -acid fire extinguisher

iii) to make bread and cake spongy

Page 41 of 135
b) How washing soda is obtained from baking soda? Write balanced chemical equation.

2 What is meant by the Chlor-alkali process? Why is it called so? Give two uses of each of the
products formed during this process.

3 Baking soda is used in small amount in making bread and cake. It helps to make these soft and
spongy. An aqueous solution of baking soda turns red litmus blue. It is also used in soda acid fire
extinguisher. Use this information to answer the following questions:
A. How does Baking Soda help to make cakes and bread soft and spongy?

B. How does it help in extinguishing fire?

C. Is the pH value of baking soda solution lesser than or greater than 7?

D. How would you show that blue color copper sulphate crystals contain water of crystallization?

4. Tabulate the following salts according to the criteria given below:

Salt pH Acid used Base used Family Formula

Calcium Chloride

Potassium Sulphate

Magnesium Chloride

Copper nitrate

5 What happens when:

(i)Excess of carbon dioxide is passed through lime water.


(ii)Dry chlorine gas is passed over slaked lime.
(iii) Electricity is passed through an aqueous solution of sodium chloride.
(iv) Gypsum is heated at 373K.
(v) A solution of sodium hydrogen carbonate is heated.

Page 42 of 135
Learning Objective Achieved Working Needs
towards reinforcement

I can identify acids and bases by


using visual and olfactory indicators.
I can illustrate reactions of acids with
metals, metallic oxides and bases.
I can express reactions of bases with
metals, nonmetallic oxides and acids.
I can explain the use of pH scale in
comparing strength of acids and
bases.
Teacher’s feedback:

Student’s feedback:

Next step in Learning:

WORK WELL DONE WORK ADEQUATELY WORK REQUIRES MORE


ATTEMPTED
PRACTICE AND EFFORT

REMARKS :

Signature: Date:

Page 43 of 135
WORKING

Page 44 of 135
WORKING

Page 45 of 135
CHAPTER -III

Synopsis:

Page 46 of 135
Chemical Properties of Metals

1) Reaction of Metals with Oxygen

Metals on burning in air form Metal Oxides


.For e.g.
a) Copper when heated in air forms Copper (II) oxide, a black oxide.

2Cu + O2 → 2CuO

(Copper) (Copper (II) oxide)

b) Aluminium forms Aluminium Oxide

4Al + 3O2 → 2Al2O3


Aluminum oxide, Zinc oxide are amphoteric oxides as they react with both acids as well as bases to
form salt and water.

For e.g.
Al2O3 + 6 HCl → 2AlCl3 + 3H2O
Al2O3 + 2NaOH → 2 NaAlO2 + H2O
Some metals oxides dissolve in water to form alkalis.
Na2O(s) + H2O (l)→ 2NaOH(aq)
K2O (s) + H2O (l)→ 2 KOH (aq)

Different metals show different reactivities towards oxygen. Metals such as a K and Na react
vigorously with air. To protect them, they are kept immersed in kerosene. At ordinary temp.surface of
metals such Mg, Al, Zn, Pb are covered with layer of oxide which prevents metals from further
oxidation.
-Fe and Cu do not burn but react with oxygen on heating to form their oxides.
-Gold and Silver do not react with oxygen even at high temp.

2) Reaction of Metals with water

Metal + H2 O → Metal oxide + Hydrogen


Soluble metal oxide + water → Metal Hydroxide
Rate of reaction of different metals is different
- Na , K react violently with cold water

2Na (s) + 2H2O(1) → 2NaOH(aq) + H2 (g) + Heat energy

2K (s) + 2H2O(l) → 2KOH (aq) + H2 (g) + Heat energy

Metals like Al, Fe, Zn do not react with cold water or hot water. They react with steam.

Page 47 of 135
2Al (s) + 3H2O(g) → Al2O3(s) + 3H2 (g)

3 Fe (s) + 4H2O(g) → Fe3O4(s) + 4H2 (g)

3) Reaction of Metals with Acids

Metal + dilute acid → Salt + Hydrogen

H2 gas is not evolved when metals react with nitric acid as it is a strong oxidizing agent.
It oxidizes H2 to produce H2O, itself gets reduced to oxides of nitrogen (N2O, NO, NO2). Mg
and Mn react with very dilute HNO3 to evolve H2 gas. The reactivity decreases in the order
Mg>Al>>Zn>Fe.
Cu does not react with dilute HCl.

4) Reaction of metals with solutions of other metal salts


Reactive metals displace less reactive metals from their compounds in solutions or molten
form.
When iron nail is kept in an aqueous Copper sulphate, the blue color of CuSO4 (aq) fades away and
changes to green in color as Fe displaces Cu from CuSO4 solution.

Eg. Fe + CuSO4 →FeSO4 + Cu

Page 48 of 135
IONIC BOND
A chemical bond formed between two ions with opposite charges. Ionicbonds form when one atom gi
ves up one or more electrons to anotheratom. These bonds can form between a metal a non-metal
and these type of bond found in salts
Eg: Sodium Chloride

Page 49 of 135
Eg: Magnesium Fluoride

Properties of ionic compounds

1. Physical nature: they are hard, crystalline solids. They are generally brittle.
2. They have high melting and boiling point.
3. Generally soluble in water and insoluble in organic solvents.
4. They conduct electricity in aqueous solution and in molten state

Extraction of Metals
Metals can be categorized into three parts on the basis of their reactivity: most reactive, medium
reactive and least reactive

Steps of Extraction of Metals

Page 50 of 135
1. Extracting metals low in the reactivity series.
➢ Ores can be reduced to metal by heating alone.(Roasting)
➢ Generally found as sulphides.
Ex: Cinnabar (HgS) : an ore of mercury
➢ Conversion of sulphide to oxide
➢ Decomposition of oxide to metal
2HgS(s) + 3O2 (g) heat→ 2HgO(s) + 2SO2 (g)
2HgO(s) heat
→ 2Hg(l) + O2(g)
Similarly copper glance (Cu2S) yields copper.
2Cu2S(s) + 3O2 (g) heat→ 2Cu2O(s) + 2SO2 (g)
2Cu2O + Cu2S heat→ 6Cu(s) + SO2 (g)
2. Extracting Metals in the middle of the reactivity series

➢ Fe, Zn, Pb, Moderately active elements.

➢ Found as sulphides or carbonates

➢ It is easier to extract metals from the oxides than from sulphides or carbonates.

a. Conversion of the carbonates and sulphides to oxides.

*Sulphide ores and carbonate ores are first converted to oxides.

• Roasting: Heating the sulphide ore strongly in the presence of the excess of
air.

2ZnS(s) + 3O2 Heat


→ 2ZnO(s) + 2SO2(g)
Page 51 of 135
• Calcination: Heating the carbonate ore strongly in the absence or limited
supply of air

ZnCO3(s) + Heat→ ZnO(s) + CO2(g)


b. Reduction of metal oxide to metal:

Metal oxide + reducing agent → Metal + oxidized reducing agent


• Reduction using carbon as reducing agent

ZnO(s) + C(s) Heat


→ Zn(s) + CO(g)

• Reduction by more active metals as reducing agent

Oxides of manganese (MnO2) and Chromium (Cr2O3) are reduced to


metal by heating with aluminium.
3MnO2(s) + Al(s) Heat→ 3Mn(l) + 2Al2O3(s) + heat
3. Extraction of metals of high reactivity:

➢ Generally found as chlorides or oxides.


➢ Metal is extracted by electrolytic reduction.
➢ The metal is obtained by electrolysis of their fused(molten)
chlorides.
➢ The metal is obtained at the cathode whereas the non-metals are
obtained at the anode.

Corrosion

The process of slow wasting away of metal by the action of atmospheric gases and moisture is
called corrosion.

Silver objects : Black deposit (Silver sulphide)


Copper objects : Green deposit (Basic copper carbonate)
Iron objects : Brown flaky deposit (Hydrated ferric oxide)
Conditions necessary for corrosion of iron.

Page 52 of 135
1. Presence of air (oxygen)
2. Presence of water
Prevention of corrosion
1. Painting
2. Oiling
3. Greasing
4. Galvanizing: Coating iron objects with a thin layer of zinc.
5. Chrome plating: Coating iron objects with chromium by electrolysis
6. Anodizing: Making layer of oxide on aluminium
7. Alloying: Making homogeneous mixtures of 2 or more metals or a metal with non-metal.

ALLOYS
An alloy is a homogeneous mixture of two or more metals or a metal and a non- metal.
Why do we make alloys: For altering the properties?
E.g.
1. Increasing hardness: Adding carbon to iron.
2. Making corrosion resistant: Adding Ni and Cr to iron. (stainless steel)
3. Lowering melting point: Mixing tin + lead. (solder)
4. Reducing electrical conductivity: Mixing Cu and Zn to make Brass
5. Reducing reactivity: Adding mercury to sodium.

6. Alloys of metal with mercury are called Amalgams

Xxxxxxxxxxxxxxxxxxx

Page 53 of 135
MULTIPLE CHOICE QUESTIONS (1 mark each)

1 An element A is soft and can be cut with a knife. This is very reactive to air and cannot be kept in open
air. It reacts vigorously with water. Identify the element from the following.
(a) Mg (b) Na
(c) P (d) Ca
2 Alloys are homogeneous mixtures of a metal with a metal or non-metal.Which among the following alloys
contain non-metal as one of its constituents.?
(a) Brass (b) Bronze
(c) Amalgam (d) Steel
3 Although metals form basic oxides, which of the following metals form an amphoteric oxide?
(a) Na (b) Al
(c) Cu (d) Fe

4 In the electrolytic reefing of copper, the electrolyte used is


(a) CuSO4 (aq) (b) Cu(OH)2
( c) Acidified ) CuSO4(aq) (d) CuSO4(s)

5 Which one of the following metals do not react with cold as well as hot water?
(a) K (b) Ca
(c) Mg (d) Fe

ASSERTION & REASON (1Mark each)


In the following questions, the assertion and reason have been put forward. Read the statements carefully
and choose the correct alternative from the following.
a. Both the assertion and reason are true, and Reason is the correct explanation of assertion.
b. Both the assertion and reason are true, but Reason is not the correct explanation of assertion.
c. Assertion is true, but reason is false.
d. Both assertion and Reason are false.
1 Assertion: Al2O3 is an amphoteric oxide.
Reason: Al2O3 react with acid as well as base to form salt and water.

2 Assertion: Nitrogen is a non-metal.


Reason: Nitrogen has 5 valence electrons

Page 54 of 135
3 Assertion: Copper does not react with Sulphuric acid.
Reason: Copper is more reactive than hydrogen.

4 Assertion: Highly reactive metals are obtained by electrolytic reduction of their molten ore.
Reason: : Highly reactive metals can be extracted by chemical reduction.

5 Assertion: Silver becomes black in colour when exposed to atmosphere.


Reason: Silver reacts with H2S gas to form Ag2S which is black in colour.

VERY SHORT ANSWER TYPE QUESTIONS (1 mark each)

1 A nonmetal X exists in two different forms Y and Z. Y is the hardest natural substance, whereas Z is a
good conductor of electricity. Identify X, Y and Z.

2 Metals generally occur in solid state. Name and write symbol of a metal that exists in liquid state at room
temperature.

3 Why do we use copper and aluminium wire for the transmission of electric current.

4 What is aqua regia?

5 A green layer is gradually formed on a copper plate left exposed to air for a week in a bathroom. What
could this green substance be?

6 Why do gallium and Cesium melt in our palm?

Page 55 of 135
. Short answer questions (SA) 2 marks each.

1 What is a thermite reaction? Give one practical application of it.

2 A solution of copper sulphate was kept in an iron pot. After a few days, the iron pot was found to have
many holes in it. Why? Also write the equation involved.

3 Give reason
i) Reactivity of aluminium decreases if it is dipped in nitric acid.
ii) Metals like Calcium and magnesium never found in their free state in nature.

4 When a metal X is treated with cold water, it gives a base Y with molecular formula XOH (Molecular
mass=40g) and liberates a gas Z which easily catches fire. Identify X, Y and Z

5 Name a metal/ non-metal:


a) Which makes iron hard and strong?
b) Which is alloyed with any other metal to make an amalgam?
C) Which is used to galvanize iron articles?
d) Whose articles when exposed to air form a green coating?
6 Why magnesium ribbon starts floating in hot water? Write the chemical equation.

. Short answer questions (SA) 3 marks each.


1 Write the electron - dot structure for sodium and chlorine atoms. How do these form a chemical bond?
Name the type of bond so formed. Why does a compound so formed have high melting point?

2 Explain in detail how copper is refined electrolytically.

Page 56 of 135
3 What is cinnabar? How is metal extracted from cinnabar? Explain briefly.

4
i) Define activity series of metals. Arrange the metals gold, copper, iron and magnesium in the order of
their increase in reactivity.
ii) What will you observe when:
(a) Some zinc pieces are put in copper sulphate solution.
(b) Some silver pieces are put into green colored ferrous sulphate solution.

5 A student was given Mn, Zn, Fe, and Cu metals Identify which of them:
a) will not displace hydrogen from dil.HCl.
b) will react only with steam to give hydrogen gas.
c) will give hydrogen gas with 5% HNO3
Write the chemical reactions involved.

A metal M has electronic configuration 2,8,3. and occurs in nature as M2O3. It is more reactive than zinc.
6 Answer the following questions:

(a) Name the metal M.


(b) Name the ore from which the metal is extracted.
(c) How is the metal oxide converted to metal?

Long answer questions (LA) 5 marks each.


1
a) Write the balanced chemical equations for the following reactions:
i) Iron metal reacts with steam
ii) Zinc sulphide is roasted
b) M is an element which is out of Cu, Fe, Al, Na. It shows the following properties:
i) One of its ore is rich in M2O3.
ii) M2O3 is not affected by water.
iii) It corrodes easily.
iv) It forms 2 chlorides - MCl2 and MCl3.
Identify M and justify your answer.

Page 57 of 135
2
Give reasons:
a) Gold and platinum are used for making jewelry.
b) Na, K and Li are stored under oil.
c) Ionic compounds have high melting points.
d) Metals are good conductors of heat.
e) Non-metals cannot displace hydrogen from dilute sulphuric acid.

3
a) Write electron dot structures for Potassium, Sulphur, calcium, oxygen, aluminium.
b) Show the formation of Na2S, CaO, and AlCl3 by the transfer of electrons.
c) What are the ions present in these compounds?

4
A metal X combine with a non- metal Y by transfer of electron to form a compound Z.
a ) State the type of bond in compound Z.
b) What can you say about the melting and boiling point of compound Z.
c) Will compound Z dissolve in kerosene or petrol?
d) Will compound Z be a good conductor of electricity? Justify your answer.

Page 58 of 135
Learning Objective Achieved Working Needs
towards reinforcement

I can understand an element as a


metal or a non-metal based on its
physical properties.
I can interpret the chemical properties
of metals and non-metals with
oxygen, water, dil acids and salt
solutions.
I can describe the ionic bonding by
electron dot structure and explain
how metals and non-metals react.
I can describe the extraction of metals
from their ores by writing balanced
chemical equations.
Teacher’s feedback:

Student’s feedback:

Next step in Learning:

WORK WELL DONE WORK ADEQUATELY WORK REQUIRES MORE


ATTEMPTED
PRACTICE AND EFFORT

REMARKS :

Signature:
Date:

Page 59 of 135
WORKING

Page 60 of 135
WORKING

Page 61 of 135
CHAPTER-4
CARBON AND ITS COMPOUNDS

Synopsis:
Carbon: Introduction
Atomic Number: 6
Electronic configuration: 2, 4
Valence electrons: 4
Property: Non-metal
Abundance: Carbon is the 4th most abundant substance in universe and 15th most abundant substance
in the earth’s crust. Compounds having carbon atoms among the components are known as carbon
compounds. Previously, carbon compounds could only be obtained from a living source; hence they
are also known as organic compounds.
Bonding in Carbon: Covalent Bond
Bond formed by sharing of electrons is called covalent bond. Two of more atoms share electrons to
make their configuration stable. In this type of bond, all the atoms have similar rights over shared
electrons. Compounds which are formed because of sharing of electrons are called COVALENT
COMPOUNDS.

Covalent bonds are of three types: Single, double and triple


covalent bond.

Versatile nature of carbon: At present almost five million organic


compounds are known to use ¦more than the elements of all the compounds put together!! The unique
nature of carbon atom and the arrangement of the bond carbon forms with other atoms enable the
existence of such a large number of organic compounds.
a) Catenation – the self- linking property of an element due to which a large number of its atoms
can be linked with each other by covalent bond is called catenation.

Page 62 of 135
b) Tetravalency- Ability of carbon atom to bond with four other atoms of carbon or atoms of some
other mono-valent element.

c) Isomerism – it is the phenomenon due to which two or more compounds have the same
molecular formula but different structural formulae.

Allotropy: Allotropy may be defined as the property of an element as a result of which it exists in
more than one forms, which are physically different and chemically similar.

Page 63 of 135
DIFFERENCES BETWEEN DIAMOND AND GRAPHITE
Graphite Diamond
1. Graphite has two dimensional 1. Diamond is a three dimensional
hexagonal planar structure tetrahedral structure.
2. Graphite is soft and greasy. 2. It is the hardest substance known.
3. It is a good conductor of heat and 3. It is the bad conductor of electricity but
electricity. good conductor of heat.
4. It is grayish black substance. 4. It is transparent.
5. Graphite is used as a lubricant. 5. Diamond is the ultimate abrasive.
6. Density of graphite is comparatively 6. Density of diamond is more
less.

Page 64 of 135
HYDROCARBONS
➢ The compounds of carbon and hydrogen in which carbon atoms are linked to each other
by single covalent bonds (C-C), are known as alkanes. These are very little reactive
chemically and are called saturated hydrocarbons.
➢ The hydrocarbons in which carbon atoms are linked to each other at least by one double
bond (C=C) or one triple bond (C ≡ C) are called as unsaturated hydrocarbons. The
unsaturated hydrocarbon with a double bond is called an alkene while those having a triple
bond are called alkynes.

Page 65 of 135
➢ Aromatics are closed chain carbon compounds e.g benzene
➢ Functional group: An atom or a group of atoms that largely determines the property of a
particular family of organic compounds is called a functional group.
➢ Homologous series is a series of similarly constituted compounds in which the members have
the same functional group, same chemical properties and any two successive members in a
particular series differ in their molecular formula by –CH2 group.

Page 66 of 135
CHEMICAL REACTIONS OF CARBON COMPOUNDS

➢ Combustion: Combustion reactions are carried by heating organic compounds with excess of
air or oxygen. Complete combustion is indicated by a blue flame while incomplete
combustion is represented by yellow flame.

➢ Oxidation – if the combustion of organic compounds is properly controlled, it may give useful
organic compounds.

➢ Addition reactions – in addition reactions, the attacking molecule (e.g Br2 or H2) is added to
the organic compound containing a double or a triple bond. The addition takes place at the
point of unsaturation and changes it to a saturated compound.

Page 67 of 135
➢ Substitution reactions: In substitution reactions, one or more atoms or groups of atoms present
in a compound are replaced or substituted by different atoms or groups

ALCOHOLS
Organic compounds, which have the hydroxyl group (–OH) attached to carbon atoms are known
as alcohols. Hydroxyl group is a functional group.

ETHANOL- C2H5OH

Page 68 of 135
Chemical properties:
a) Reaction with sodium metal:

b) Reaction with carboxylic acids (esterification): alcohols react with carboxylic acids in presence
of conc.H2SO4 as catalyst to form esters which have sweet fruity smell.
C2H5OH + CH3COOH → CH3COOC2H5 + H2O
conc.H2SO4

Page 69 of 135
In cold countries ethanol is used as anti-freeze and is an
efficient fuel.

CARBOXYLIC ACID

ETHANOIC ACID

Page 70 of 135
➢ Dilute solution (5-8%) of ethanoic acid in water is known as vinegar. It is used for preserving
food.
Esterification: The compound formed by the reaction of an alcohol with a carboxylic acid in
presence of sulphuric acid is known as ester, and the reaction as esterification. Esters have a
very sweet fruity smell. Naturally occurring esters are found in fruits. They are used for
making perfumes
➢ Saponification: The hydrolysis of an ester in presence of an alkali or an acid.
Hydrolysis with an alkali is called saponification.

SOAPS AND DETERGENTS:

Page 71 of 135
➢ Soaps have two main parts – a polar head which is hydrophilic & a non-polar hydrocarbon
tail which is hydrophobic.
➢ Soaps are ineffective in hard water as they react with Ca+2 and Mg+2 ions in hard water to
form insoluble scum.

Detergents can be used in hard water also as the calcium and magnesium salts produced are soluble in
water
SOAPS DETERGENTS
1. Soaps are sodium salts of long chain fatty 1. Synthetic detergents are sodium salts of long
acids chain alkyl sulphonates.
2. They cannot be used in hard water as they 2. They can be used in hard water as they do not
produce precipitates with Ca and Mg ions form precipitate with Ca+2 and Mg+2 ions.
+2 +2

of hard water. 3. They are non-biodegradable, hence cause


3. They are bio-degradable. Therefore, they pollution of water sources.
do not cause any pollution.

NOTE:
1. While preparing soap a small amount of common salt is added to the reaction mixture of vegetable
oil and sodium hydroxide. Common salt lowers the solubility of soap and helps in precipitation. So it
is easy to separate soap from the remaining solution.

2. If a straight chain hydrocarbon is used in the detergent instead of a branched chain hydrocarbon,
then the detergent becomes biodegradable. Thus the major disadvantage of detergents can be
overcome by reducing the branching in the carbon chain.

xxxxxxxxxxx

Page 72 of 135
EXERCISE

MULTIPLE CHOICE QUESTIONS (1Mark each)

1 Which of the following substance is added to denature Ethanol?


a) methanol
b ) pyridine
c) copper sulphate
d) all of them

2 Which of the following represents the correct decreasing order of hydrogen atoms ?
a) alkanes, alkenes , alkynes
b) alkanes, alkynes, alkenes
c) alkenes, alkynes , alkanes
d ) alkynes , alkanes , alkenes

3 Detergents are sodium or potassium salts of long chain of ;-


a) aldehydes
b) ketones
c) carboxylic acid
d) sulphonic acid

4 . Which of the following salt when dissolved in water produce hard water?
a) calcium sulphate
b) magnesium bicarbonate
c) calcium chloride
d) any of the above

5 . Which of the following is not an allotropic form of carbon?


a) fluorine
b) fullerene
c) diamond
d) graphite

ASSERTION & REASON (1 mark each)


In the following questions, the assertion and reason have been put forward. Read the statements
carefully and choose the correct alternative from the following.
a. Both the assertion and reason are true, and Reason is the correct explanation of assertion.
b. Both the assertion and reason are true, but Reason is not the correct explanation of assertion.
c. Assertion is true, but reason is false.
d. Both assertion and Reason are false.
1
Assertion: Chemical bonds in organic compounds are covalent in nature.
Reason: Covalent bond is formed by the sharing of electrons in the bonding atoms.

Page 73 of 135
2 . Assertion: Diamond is the hardest crystalline form of carbon.
Reason: Carbon atoms in diamond are arranged in tetrahedral nature.

3 Assertion: Due to catenation a large number of carbon compounds are formed.


Reason: Carbon compounds show the property of allotropy.

4 . Assertion: Even though it is a non-metal, graphite conducts electricity.


Reason: It is due to the presence of free electrons

5 . Assertion: Acetic acid is heated with Na2CO3 in a test tube. A colourless and odourless gas (X) is evolved.
The gas turns lime water milky.
Reason: The gas evolved is hydrogen.

VERY SHORT ANSWER TYPE QUESTIONS (1 MARK EACH)

1. Write the general formula of hydrocarbon alkene .Write the name of simplest alkene.

2. Why do covalent molecules have low melting and boiling points?

3. Name the reaction which takes place when ethanoic acid reacts with ethanol. What is the general
name of the product obtained in this reaction?
4. Butanone is a four carbon per molecule compound. Name the functional group present in it
and write its structure..
5. Write the molecular formulae of an alkane and an alkene with 20 carbon atoms.

SHORT ANSWER TYPE QUESTIONS (2 Mark each)

6. How can vegetable oils be converted to saturated fat? Give the reaction along with
conditions.
7. Which of the following will have a double bond between two carbon atoms:
C2H2, C4H8, C4H6, C3H8.
8. What would be observed on adding a 5% solution of alkaline potassium permanganate drop
by drop to some warm ethanol taken in a test – tube? Write the name of the compound formed and
also write a balanced chemical equation for the reaction involved.
9. Write the structural formula of the following:
a) Chloroethane
b) Propanol
10. A compound X has molecular formula C3H4. One mole of X reacts with 2 moles of hydrogen to yield a
compound Y. Deduce the structures of X and Y.

Page 74 of 135
The questions given below carry 3 marks each.
11. Give reasons why:
• Carbon compounds are usually poor conductors of electricity?
• The use of detergents can cause serious environmental problem.
• Mixture of ethyne and oxygen is burnt for welding and not ethyne and air.
12. Define esterification and saponification. What is the difference between the two reactions?
Give one industrial use of each of the reactions.
13. Identify the compound given below and write its IUPAC name. Name the constituents from
which this compound is prepared and write a balanced chemical equation for the preparation.
H O
│ ║
H – C – C - O –C2H5

H
14 Why do some hydrocarbons burn with a sooty flame? Which of the following ethyne,
methane, propene, will give the maximum smoky flame? Give reason for your answer.

15. What is isomerism? Write the possible isomers for a hydrocarbon having molecular formula
C5H12 and give their names.
The questions given below carry 5 marks each.

16. An organic compound ‘A’ is widely used as a preservative in pickles and has a molecular
formula C2H4O2. This compound reacts with ethanol to form a sweet smelling compound
‘B.
(i) Identify the compound ‘A’
(ii) Write the chemical equation for its reaction with ethanol to form compound ‘B’.
(iii) How can we get compound ‘A’ back from ‘B’?
(iv) Name the process and write corresponding chemical equation.
(v) Which gas is produced when compound ‘A’ reacts with washing soda?
17. How will you prepare the following?
a. ethane from ethanol
b. chloromethane from methane
c. propane from propene
d. ethanoic acid from ethanol
e. Propanol to Propanoic acid.

Page 75 of 135
18. An organic compound A on heating with concentrated H2SO4 forms compound B which
on addition of one mole of hydrogen in presence of Ni forms compound C. One mole of
compound C on combustion forms 2 moles of a gas D which turns lime water milky and
3 moles of water. Identify the compounds A, B, C and D and write the chemical
equations of reactions involved
19. A compound X of molecular formula C2H4O2 reacts with sodium metal to form
compound Y and a gas Z which burns with a pop sound. Compound X on treatment with
compound P in presence of an acid gives a sweet-smelling liquid Q. On addition of
NaOH to X, it also forms Y and water. Q on treatment with NaOH gives back Y and P.
Identify X, Y, Z, P and Q. Write the balanced chemical equation for all the reactions
involved.
20. a) Define Functional group. Give examples of four different functional groups with 3
carbon atoms.
b) Draw the possible isomers of the compound with molecular formula C3H6O and
also give their electron dot structures.

Page 76 of 135
WORKING

Page 77 of 135
Learning Objective Achieved Working Needs
towards reinforcement

I can understand and describe


covalent bonding in carbon
compounds.
I can recall the versatile nature of
carbon and study about its allotropes.
I know the difference between
saturated and unsaturated
hydrocarbons.
I can explain the nomenclature and
chemical properties of carbon
compounds.
Teacher’s feedback:

Student’s feedback:

Next step in Learning:

WORK WELL DONE WORK ADEQUATELY WORK REQUIRES MORE


ATTEMPTED
PRACTICE AND EFFORT

REMARKS :

Signature: Date:

Page 78 of 135
CHAPTER 5
PERIODIC CLASSIFICATION OF ELEMENTS

Synopsis:

To make the study of the elements, easy and systematic, elements are classified.

Doberiener’s Triads (Law of triads)

Elements with similar properties were arranged in groups of three in increasing order of atomic
masses and it was found that the atomic mass and properties of the middle element were
approximately an average of the first and third.

Example:

Triad Atomic mass

Li 6.9
Na 23.0 Atomic Mass of Na = 6.9+39=22.95
K 39.0 2

Limitation: Only three triads could be successfully being obtained.


(Li, Na, K) (Ca, Sr, Ba) (Cl, Br, I)

Newland’s Law of Octaves

When elements are arranged in increasing order of atomic mass, every eighth element had properties
similar to the first element.
This resembles the notes of music therefore it was called Law of octaves.
All elements (56 elements) discovered at that time could not be arranged in this manner.

Limitation: It was applicable for elements up to Ca. Co and Ni placed in the same slot as F,Cl and Br
have entirely different properties. Fe which is similar to Co and Ni is placed in a separate place.
New elements discovered later could not be fit into the table.

Mendeleev’s Classification:

Mendeleev’s Periodic Law states that physical and chemical properties of elements are the periodic
function of their atomic masses.

Page 79 of 135
Mendeleev studied in detail the properties of elements, the formation of their oxides and
hydrides, and arranged them in order of increasing atomic masses.

Important features of Mendeleev’s table:


• He classified 63 elements discovered at that time.
• The vertical columns were called groups. His table had 8 groups. Groups from I to VII
were subdivided into A and B. VIII group was not subdivided, it had three elements in
each period.
• The horizontal rows were known as periods. His table had 6 periods.

Advantages of Mendeleev’s table:


• He could classify all the elements known at that time (63).
• He left gaps in the table and predicted properties of the elements which were discovered later.
eka-Aluminium Gallium
eka-Boron Scandium
eka- Silicon Germanium
It helped in the correction of atomic masses of some elements on the basis of their position.
• When noble gases were discovered later they, were kept in the separate group without
disturbing the table.

Limitations of Mendeleev’s Table:


• No fixed position could be given to hydrogen as it shows resemblance with alkali metals in its
reactions with halogens, oxygen and sulphur. Hydrogen exists as a diatomic molecule like
halogens and forms covalent compounds with metals and non-metals like halogens.
• Though most elements were in order of increasing atomic masses there were some cases
where the order was not maintained, but similarity in properties was maintained. (Co=58.93
and Ni=58.71)
• Atomic masses do not increase in a regular manner therefore number of elements to be
discovered between two elements cannot be predicted.
• Mendeleev’s table could not give a place to isotopes because their masses are different but
properties are same.

MODERN PERIODIC TABLE


Modern Periodic Law:
The physical and chemical properties of elements are a periodic function of their atomic
numbers.

Advantages of Modern Periodic table:


• When elements are arranged in increasing order of atomic numbers, all anomalies of
Mendeleev’s table were corrected. Position of hydrogen however, still could not be fixed.
• Isotopes of an element have same number of protons, i.e. same atomic number, hence they can
be placed at one position in the periodic table.
Page 80 of 135
• The anomaly of elements with higher atomic mass before that of lower atomic mass was
removed as the modern table was based on increasing atomic numbers.
• Knowing the atomic number of an element, its position in the periodic table can be
determined and properties predicted.

Characteristics of periods (the horizontal rows are called periods.)


There are 7 periods.
1st period - very short period, consists of only 2 elements.
2nd&3rd periods - short periods, consisting of 8 elements each.
4th&5th periods - long periods, consisting of 18 elements each.
6th period - very long period, consisting of 32 elements.
7th period – very long period, consisting of 32 elements .
• The number of elements in a period depend upon the maximum number of electrons which
can be accommodated in a shell (2n2 rule), where n is the number of the shell.
• The first element of any period has 1 valence electron and the last, the inert gas, has 8
electrons. There is an increase of one valence electron as atomic number increases by one on
moving from left to right in a period.
• The number of shells remains same in elements of a period. The number of shells gives the
number of the period to which the element belongs.

Characteristics of groups (vertical columns are known as groups).


There are 18 vertical columns called groups numbered from 1 to 18.

• The outermost shell of each element in a group have the same number of valence electrons i.e.
same outer shell electron configuration.
• The number of shells increases as we go down the group.

Periodicity in Properties (Trends in the modern periodic table)


Valency:
It is the combining capacity of an element, i.e. the number of electrons an atom can lose, gain or
share to form a bond.

Valency can be determined by the number of electrons in the outermost shell of the atom.

Variation in a group: Valency of all elements in a group remains same.

Variation in a period: Valency increases from 1 to 4 and then decreases to 1, on moving from left to
right in a period. It is zero for noble gases.

Atomic Size:
It is measured as radius of an atom. Atomic radius is the distance from the center of the nucleus to
the outermost shell of an isolated gaseous atom. The unit is pm (picometre 1pm=10-12m).

Page 81 of 135
Variation in a group: Atomic radius increases from top to bottom in a group as a new electron shell
is added for each subsequent element.

Variation in a period: Atomic radius decreases from left to right along a period because the net
nuclear charge increases for every next element; hence the outer shell electrons are attracted closer to
the nucleus.
Metallic and Non-metallic Character:

Metals are placed on the left and in the center while nonmetals are on the right side of the periodic
table.
Metals and non-metals are separated diagonally by metalloids or semi metals, from Boron to
Astatine.

Metalloids have properties intermediate between metals and non-metals, eg. B, Si, Ge, As, Sb etc.

Metallic character.
Metals have 1 to 3 electrons in their outermost shells. They lose electrons to give positive ions, are
electropositive in nature.
• Metallic character, i.e. the tendency to loose electrons, decreases along a period as effective
nuclear charge increases.
• Metallic character increases down the group as effective nuclear charge on valence electrons
decreases due to addition of more shells.
In general metals form basic oxides.

Non-metallic character:
Non-metals have 4 to 8 electrons in their outermost shells. They share or gain electrons. They are
electronegative in nature.
• Nonmetallic character increases along a period as the size of atom decreases and an electron
can be gained with greater ease.
• Nonmetallic character decreases down the group as size of atom increases and thus
attraction of nucleus on gained electron is reduced. These properties decrease down the group.
In general nonmetals form acidic oxides.

Page 82 of 135
A. MULTIPLE CHOICE QUESTIONS (1 mark each).

1 . Elements in the same vertical group of the periodic table have same
A. Number of valence electrons
B. Atomic number
C. Atomic mass
D. Atomic volume

2 . Which of the following always increases on going from top to bottom in a group?
A. Metallic character
B. Electronegativity
C. Oxidizing power
D. Tendency to get reduced

3 . In modern periodic table all the elements are arranged in ascending order of
A. Valency
B. Atomic mass
C. Atomic number
D. Valence electrons

4 Across period atomic size decreases due to


A. Shielding effect
B. Photoelectric effect
C. Increase in nuclear force of attraction
D. Decrease in nuclear force of attraction

5 .Ability of atom to attract electrons towards itself is called


A. electron affinity
B. electronegativity
C. ionization energy
D. shielding effect

ASSERTION & REASON QUESTIONS (1 mark each)


In the following questions, the assertion and reason have been put forward. Read the statements
carefully and choose the correct alternative from the following.
a. Both the assertion and reason are true, and Reason is the correct explanation of assertion.
b. Both the assertion and reason are true, but Reason is not the correct explanation of assertion.
c. Assertion is true, but reason is false.
d. Both assertion and Reason are false.

Page 83 of 135
1 Assertion: The elements of the same group have similar chemical properties.
Reason: The elements of the same group have the different number of valence electronic.

2 Assertion: Elements of group 16 are monovalent.


Reason: Elements of group 16 have seven electrons in their outermost/valence shell.

3
Assertion: Silicon is a metalloid.
Reason: Silicon shows only properties of non-metals.

4 Assertion: Sixth and seventh periods in the periodic table contains 14 elements.
Reason: In the periodic table, 14 elements of sixth and seventh periods are known as lanthanoids
and actinoids
5 Assertion: Mendeleev’s arranged elements in horizontal rows and vertical coloumns.
Reason: Mendeleev’s ignored the order of atomic weight thinking that the atomic measurements
might be incorrect.

VERY SHORT ANSWER TYPE QUESTIONS (1Mark each)

1 Write the number of horizontal rows in the modern periodic table. What are these rows called?

Out of Li and K , which has stronger metallic character and why ?


2

3
An element has atomic number 17, to which group a nd period does it belong?

4 P, Q, & R are three elements in Dobereiner’s triad. If the atomic mass of P=35.5 and that of
R=127, calculate the atomic mass of Q.

Name two elements whose properties were predicted based on their position in Mendeleev’s
periodic table.

Page 84 of 135
6 Which of the following exhibits maximum number of valence electrons?

Na, Al, Si, P

. Short answer questions (SA) 2 marks each.

1 An element X has atomic number 12.

a) Write its electronic configuration. b) State the group to which X belongs

c) Is X a metal or non-metal? d) Write the formula of its oxide.

2
On which side of the periodic table are the metals placed? What is the name given to group 1 and 2
elements?

3
What were the two achievements of Mendeleev’s Periodic table?

4 Why do all the elements of the (a) same group have similar properties, (b) same period have
different properties ?

5
State Mendeleev’s periodic law. What was the basis of classification of elements in this table?

6 How does metallic character of elements change along a period and in a group in the modern
periodic table? Give reason.

. Short answer questions (SA) 3 marks each.

1
What were the drawbacks of Mendeleev’s periodic table? How were they overcome in the modern
periodic table? State two examples to justify the position of elements in modern periodic table.

Page 85 of 135
2 The atomic radii of three elements X, Y and Z of a period of the periodic table are 186pm; 104pm
and 143pm respectively. Giving a reason, arrange these elements in the increasing order of atomic
numbers in the period.

3 Based on electronic configuration how will you identify


(i) Chemically similar elements
(ii) The first element of a period
(iii) Metallic/non-metallic nature
4 An element X has mass number 35 and number of neutrons 18. Write the atomic number and
electronic configuration of X. Also write group number, period number and valency of X.

Two elements X and Y have atomic numbers 12 and 16 respectively. Write the electronic
5 configuration for these elements. To which period of the modern periodic table do these two
elements belong? What type of bond will be formed between them and why?

6 An element X belongs to the 3rd period and group-2 of the periodic table. State:
a) Atomic number
b) Electronic configuration.
c) Whether it is a metal or a non-metal _
7 An atom has the electron arrangement as 2, 8, 7.
(i) What is the atomic number of this atom?
(ii) To which of the following will it be chemically similar, 7N, 9F, 15P and 18Ar?
Give reason why you expect the similarity.
8
The atoms A & B have electronic configuration (2,8,8,2) and (2,6) respectively.
a) To which periods do A & B belong?
b) To which groups do A & B belong?
c) Write the formula of the compound formed between A and B. Will it dissolve in water
or conduct electricity?
Long answer questions (LA) 5 marks each.

1 An element X belongs to group 2 and another element Y belongs to group 16 of the periodic table.
(i) What is the number of valence electrons in X and Y?
(ii) What are the valencies of X and Y?
(iii)What is the nature of the compound formed by them?
(iv) Give the formula of the compound formed.
(v) State whether the compound formed will conduct electricity or not?

Page 86 of 135
2 An element X which is a yellow solid at room temperature shows catenation and allotropy to a
small extent. X form two oxides which are also formed during the thermal decomposition of
ferrous sulphate crystals and are the major air pollutants.
(a) Identify the element X.
(b) Write the electronic configuration of X.
(c) Locate the position of the element in group and period in the Modern Periodic Table.

(d) Write the names and formulae of the oxides formed on thermal decomposition of FeSO4?
(e) What would be the nature (acidic/ basic) of oxides formed?

a) Name three elements of group-17, what is the name of this group of elements?
3 b) Name two alkaline earth metals. To which group do they belong?
c) Name four alkali metals. To which group do they belong?
d) The three elements predicted by Mendeleev from the gaps in his periodic table which were
known as Eka- boron, Eka- aluminium and Eka silicon, but were discovered later.
e) Group 18 elements are also called as inert gases or zero group elements. Why are they
called so?

4 The following table shows the position of six elements A, B, C, D, E and F in the periodic table.

Using the above table answer the following questions :


(a) Which element will form only covalent compounds?
(b) Which element is a metal with valency 2?
(c) Which element is a non-metal with valency of 3?
(d) Out of D and E, which one has a bigger atomic radius and why?
(e) Write a common name for the family of elements C and F.

5 An element X (2,8,2) combines separately with NO31- and (SO4)2 -, (PO4)3 - radicals. Write the
formulae of the three compounds so formed. To which group of the periodic table does the element
‘X’ belong? Will it form covalent or ionic compound? Why?

Page 87 of 135
WORKING

Page 88 of 135
Learning Objective Achieved Working Needs
towards reinforcement

I can recall Doberiner’s Law of


triad’s and Newton’s Law of Octaves.
I can interpret the way in which
elements are arranged in Mendeleev’s
Periodic Table
I can describe the position of the
elements in the Modern Periodic
Table.
I can describe the trends in the
Modern Periodic Table
Teacher’s feedback:

Student’s feedback:

Next step in Learning:

WORK WELL DONE WORK ADEQUATELY WORK REQUIRES MORE


ATTEMPTED
PRACTICE AND EFFORT

REMARKS :

Signature: Date:

Page 89 of 135
PRACTICALS
EXPERIMENT NO.1

Experiment 1A

AIM: To perform and observe the following reactions and classify them into:

i) Combination Reaction
ii) Decomposition Reaction
iii) Displacement Reaction
iv) Double Displacement Reaction

1. Action of water on quick lime.


2. Action of heat on Ferrous Sulphate crystals
3. Iron Nails kept in copper sulphate solution
4. Reaction between Sodium sulphate and Barium chloride solutions

MATERIALS REQUIRED: Quick lime (calcium oxide),water,

beaker THEORY:

Combination Reaction: A reaction in which two or more substances combine to form a new substance
is called combination reaction. Calcium oxide reacts vigorously with water to form calcium
hydroxide (slaked lime) with the evolution of heat.

CaO(s) + H2O(l) → Ca(OH)2 (aq) + Heat

Such reactions in which heat is evolved are called exothermic reactions.

OBSERVATION TABLE:

EXPERIMENT OBSERVATION INFERENCE


Take 5g of Calcium A vigorous reaction takes Since Calcium oxide and water
oxide in a beaker and place to form slaked lime react to form a single product
add water to it slowly. and the beaker in which the i.e slaked lime,it is a
reaction is carried out combination reaction.Also it is
becomes hot. an exothermic process since
heat is evolved.

RESULT: The reaction of calcium oxide with water is a combination reaction

Page 90 of 135
Experiment 1B

MATERIALS REQUIRED: Ferrous sulphate crystals, dry test tube, burner

THEORY:
Decomposition reaction: A reaction in which a single compound breaks down to produce two or
more simpler substances is called decomposition reaction. When green coloured ferrous sulphate
crystals(FeSO4.7H2O) are heated, they first lose water and the colour changes to form anhydrous
FeSO4
Heat
FeSO4.7H2O (s) FeSO4 (s) + 7 H2O (l)

This on further heating gives out a characteristic smell of burning sulphur leaving behind a reddish
brown residue of ferric oxide.

2 FeSO4 (s) Fe2O3 (s) + SO2 (g) + SO3


Ferrous sulphate Ferric oxide sulphur dioxide sulphur trioxide
(green) (Reddish brown) (having smell of
burningsulphur)

OBSERVATION TABLE:

EXPERIMENT OBSERVATION INFERENCE


Take a small amount of The green colour of ferrous Ferrous sulphate decomposes
ferrous sulphate crystals sulphate crystals changes to into simpler compounds Fe2O3,
in a dry test tube and heat reddish brown and smell of SO2 and SO3 on heating.
it strongly over a flame. burning sulphur is obtained.

RESULT: Heating of ferrous sulphate is a thermal decomposition reaction.

Experiment 1C

Materials Required: Iron nail, copper sulphate, distilled water.


Theory:
➢ Iron is more reactive than copper as it is above copper in the reactivity series.
➢ Displacement reaction: It is a reaction in which a more reactive metal displaces a less reactive
metal from its salt solution.
Reaction:
Fe + CuSO4 FeSO4 + Cu

(blue) (green)
Page 91 of 135
S.No Experiment Observation Inference
1 Clean an iron nail and After 15 minutes the The Brown coating on the
put it a test tube. Add colour of the solution
about 10 ml of copper changes from blue to light nail shows that copper from
sulphate solution to it green and a brown coating
and leave it for some is observed on the surface the copper sulphate solution
time. of the nail.
has been displaced by iron

and green coloured

Ferrous sulphate has been


formed. Iron is more reactive
than copper

RESULT: The reaction of Iron with copper sulphate is a displacement reaction.

Experiment 1D

Aim: Sodium sulphate with Barium chloride in the form of their solutions in water.

Materials Required: Sodium sulphate solution, barium chloride solution, conical flask and glass rod.
Theory:
Double displacement reaction: A reaction in which exchange of two metal atoms in the given
solutions takes place simultaneously, e.g.

BaCl2(aq) + Na2SO4(aq) BaSO4(s) + 2NaCl(aq)


(white ppt)

OBSERVATION TABLE:

S.No. Experiment Observation Inference


1 Take 10 ml each of sodium A white A double displacement reaction
sulphate and barium precipitate is takes place. A white precipitate of
chloride solutions in formed barium sulphate is formed and
different test tubes. Mix sodium chloride remains in solution
the two solutions.

RESULT: The reaction of sodium sulphate with barium chloride is a double displacement reaction

Page 92 of 135
PRACTICAL BASED QUESTIONS:
1. What happens when an iron nail is dipped in copper sulphate solution. Write two
observations and a balanced chemical equation.
Ans. As iron is more reactive than copper it displaces copper and color of the solution fades.
Blue solutions changes to pale green and reddish brown copper metal deposits at the bottom.
CuSO4 + Fe ------------FeSO4 + Cu

2. What happens when zinc reacts with dil sulphuric acid? Write the equation and name the
reaction.
Ans. Bubbles of hydrogen gas will be evolved.
Zn + H2SO4 → ZnSO4 + H2
This is a single displacement reaction.
3. Name the white ash formed on burning Mg ribbon. Is it acidic or basic? Write the chemical
equations to justify your answer.
Ans. Magnesium oxide
This is basic in nature as red litmus turns blue on mixing with water.
MgO + H2O → Mg(OH)2.
4. What happens when barium chloride and sodium sulphate are mixed together. Name the type of
chemical reaction.
Ans. White ppt of Barium sulphate is formed.
The reaction is double displacement reaction.

5. Heating of lead nitrate gives brown fumes.


Write a balanced chemical equation. Name the substance with brown fumes.
What is the color of lead nitrate?
Ans. 2 Pb(NO3)2 (s) → 2 PbO (s) + 4 NO2 (g) + O2 (g)
NO2 is a brown gas.
Lead nitrate is white in color.
6. What is the color change when blue copper sulphate is heated strongly? Write the chemical
reaction involved.
Ans. Color changes from blue to white as they lose water molecules.
CuSO4.5H2O → CuSO4 + 5H2O

7. What happens when Silver chloride is exposed to sunlight? Write the color change? Define the
type of the reaction.
Ans. Silver chloride when exposed to sunlight decomposes to form grey Silver and chlorine gas,
color changes from white to grey.
Decomposition reaction is when a single reactant breaks into two or more products by means of heat,
light and electricity.

Page 93 of 135
8. Name two metals which can displace hydrogen from acids. Name two metals which cannot
displace hydrogen from acids.
Ans. Na and K
Cu and Hg

9. Why do we add a few drops of sulphuric acid during electrolysis of water?


Name the gases evolved at cathode and anode.
Ans. Sulphuric acid increases the conductivity of the water as distilled water is a bad conductor of
electricity.
O2 gas is evolved at anode and H2 gas at cathode.

10. Write the chemical equation when barium chloride is mixed with sodium sulphate? Define the
type of this reaction.
Ans. BaCl2 + Na2SO4 → 2NaCl + Ba SO4
A double displacement reaction takes place when exchange of radicals takes place.

---------------------------------------------------------------------------------

Page 94 of 135
EXPERIMENT NO. 2A

Objective: To carry out the reactions of an acid (HCl) with (i) litmus solution (blue or red), (ii) zinc
metal, (iii) sodium .carbonate

Materials required: Boiling test tube, test tube holder, Hydrochloric acid, litmus solutions (blue and red),
zinc metal, sodium carbonate and distilled water.

Theory:
HCl turns blue litmus solution red and it does not affect a red litmus solution.
HCl reacts with zinc metal liberating hydrogen gas.
Zn(s) +2 HCl(aq) → ZnCl2(aq) + H2(g)
Hydrogen gas burns in oxygen with a pop sound.
2H2(g) + O2(g) → 2H2O(l)
HCl reacts with sodium carbonate liberating carbon dioxide gas.
Na2CO3(s) + 2HCl (aq) → 2NaCl(aq) + H2O(l) + CO2(g)
Carbon dioxide gas turns lime water milky due to the formation of calcium carbonate.
Ca(OH)2 + CO2 → CaCO3 + H2O

Procedure and observation table:

S.No.
Experiment Observation Inference
1 Litmus solution:
a) Take HCl in a test tube Blue litmus turns red. HCl is acidic in
and add few drops of nature.
blue litmus solution.

b) Take HCl in a test tube No change in the colour of


and add few drops of red the red litmus solution. Acids do not affect
litmus solution. red litmus solution
and this shows that
HCl is an acid.
2 Zinc metal:
Add zinc metal into a test Gas is evolved. Zinc reacts
tube containing HCl. vigorously with HCl
forming hydrogen
gas.

Page 95 of 135
3 Sodium carbonate:
a) Take sodium carbonate Brisk effervescence is When sodium
in a dry test tube and add evolved. carbonate reacts with
slowly few drops of HCl. HCl it produces CO2.

b) Pass the gas through CO2 reacts with


limewater. Limewater turns milky. limewater to form
calcium carbonate.

Precaution:

i) HCl should be handled with care.


ii) Small quantities of Zn and HCl should be used for the reaction, otherwise large amount of
hydrogen gas will be produced which may cause explosion.

EXPERIMENT NO.2B

Objective: To carry out the reactions of a base (NaOH) with (i) litmus solution (blue or red), (ii)
Zinc metal, (iii)Sodium carbonate.

Materials required: Boiling test tube, test tube holder, sodium hydroxide solution, litmus solutions
(blue and red), zinc granules, sodium carbonate and distilled water.

Theory:
• NaOH turns red litmus solution blue and it does not affect a blue litmus solution.
• NaOH reacts with zinc metal liberating hydrogen gas.
Zn(s) + 2NaOH(aq) → Na2ZnO2(aq) + H2(g)

Page 96 of 135
Procedure and observation table:

S.No.
Experiment Observation Inference
1 Litmus solution:
a)TakeNaOH in a test tube No change in the NaOH is a base and
and add few drops of blue colour of the blue does not affect blue
litmus solution. litmus solution. litmus solution.

Red litmus turns blue. NaOH is alkaline in


b)TakeNaOH in a test tube nature.
and add few drops of red
litmus solution.
2 Zincmetal:
Add zinc metal into a test tube Gas is evolved. NaOH reacts with
containing NaOH and heat it. zinc forming
hydrogen gas.

3 Sodium carbonate:
a) Take sodium carbonate No reaction Sodium hydroxide is
in a dry test tube and add an alkali. It does not
slowly few drops of react with sodium
NaOH . Carbonatebecause
both will mix to give
basic solutions.

Precautions: i) Small quantity of chemicals should be used.


ii) Handle NaOH carefully because it can burn skin and clothes.

Page 97 of 135
PRACTICAL BASED QUESTIONS:

1. A student dropped few pieces of marble in dilute hydrochloric acid contained in a test tube.
The evolved gas was then passed through lime water. What change would be observed in
lime water? Write balanced chemical equation for both the change observed?

Ans.When dilute HCl is added to marble which is calcium carbonate, it forms calcium chloride,
water and carbon dioxide. The chemical equation for the reaction is as follows:

CaCO3 + 2HCl → CaCl2 + H2O + CO2

Carbon dioxide gas turns limewater milky due to the formation of calcium carbonate.The chemical
equation showing reaction between lime water and carbon dioxide is as
follows: Ca(OH)2 + CO2 → CaCO3 + H2O

2. What happens when a small amount of copper oxide is mixed with dilute hydrochloric acid?
Indicate the color change.
Ans. It forms blue green copper chloride. As copper oxide is a basic oxide, it reacts with an
acid to form salt and water.

3. Two solutions of A and B have pH values of 5 and 8. Which solution will be basic in nature?
Which solution neutralize a base?

Ans. Solution B is basic in nature. Solution A is acidic so it will neutralize a base.

4. How will you test for a gas, which is liberated when HCL reacts with an active metal?
Hydrogen gas is liberated.

Ans. Bring a burning matchstick near the gas. The flame extinguishes with a pop sound.

5. What would be the color of litmus in a solution of sodium carbonate? Give reason for your
answer.
Ans. Blue.

Sodium carbonate is a basic salt as it is made up of strong base and a weak acid.

6. Name three metals which do not react either with cold or hot water but they react with
steam to form hydrogen gas. Write the chemical equation with any one of these metals.

Page 98 of 135
Ans. Al, Fe and Zn
2Al + 3H2O → Al2O3 + 3H2

7. Why the lime water turns milky after passing CO2 gas through it. Why the milky lime water
turns colorless when excess of CO2 is passed through it?
Write the chemical equations involved.

Ans. When CO2 gas passes through lime water it turns milky due to the formation of
insoluble Calcium carbonate. After passing excess of CO2 gas, It forms Calcium bicarbonate
which is soluble and forms a colorless solution.

Ca(OH)2 + CO2 → CaCO3 + H2O

CaCO3 + H2O +CO2 → Ca(HCO3)2

8. Why does the bulb glow when we pass electric current through a salt solution? Why the bulb
does not glow when we pass current through a sugar solution?
Ans. Salt solution form ions thus able to conduct electricity but sugar solution doesn’t form
ions , so no current flows and the bulb doesn’t glow.

9. While diluting an acid, why is it recommended that the acid should be added to water and not
vice versa?
Ans. Acid should be added to water slowly dropwise with constant stirring because it is a
highly exothermic reaction . It evolves a lot of heat which may splash out and burn us if water
is added to acid.

10. Why HCl gas doesn’t show change in the dry blue litmus paper but show color change in the
moist blue litmus paper?
Ans. HCl gas doesn’t ionize in presence of dry litmus paper. So due to the absence of H+ ions
it doesn’t behave like an acid and thus shows no color change with blue litmus paper.
On the other hand, HCl gas ionizes in contact with water present on the moist blue litmus paper. Thus
due to the presence of H+ ions it behaves as an acid and turns moist blue litmus paper into red.

Page 99 of 135
EXPERIMENT NO.3

Objective: To find the pH of the given samples of solids/liquids/ juices using pH paper/universal
indicator.

Materials Required: Lemon juice, dil. Sodium hydroxide (NaOH), dil.hydrochloricacid(HCl),


dil.solution of sodium bicarbonate(NaHCO3), distilled water, dil.ethanoic acid solution(CH3COOH),
pH papers and standard colour chart to compare pH value, white tile and fine droppers.

Theory:
• pH is a measure of the hydrogen ion [H+] concentration solution of a solution.
• The pH of a solution tells us how acidic or alkaline it is. The pH scale runs from 0 to 14.
• A pH 7 shows that the solution is neutral-neither acidic nor alkaline.
• A pH of less than 7 indicates acidity, and the lower the value the greater the acidity.
• A pH of more than 7 indicates alkalinity, and the higher the value higher the alkalinity.

Procedure: i) Take 6 strips of pH paper and place them on a tile. This strip of pH paper is called test
strip.
ii) Place a drop of the test solution on the strip of the pH paper with the help of a fine
dropper.
iii) Observe the colour produced and match it with the different colour shades of the
standard colour pH chart.
iv) Note down the pH from the colour chart, of that colour which matches the most with
colour produced on the pH paper.
v) Find out the pH values of all the samples given to you.

Observation table and inference:

S.No. Sample solution Colour produced on pH Approximate pH(from Inference


paper chart)
1 Lemon juice Acidic
2 DilHCl Acidic
3 DilNaOH Basic
4 Baking soda Basic
solution
5 Water Neutral
6 Ethanoic acid Acidic
Result: In the given samples we have observed that:
• The pH values of lemon juice, dil. hydrochloric acid(HCl) and dil.ethanoic acid
solution(CH3COOH) is less than 7.Therefore these have acidic character.
• The pH values of dil sodium hydroxide (NaOH) and dil. solution of sodium bicarbonate
(NaHCO3) is more than 7. Therefore these have basic character.
• The pH value of distilled water is 7. Therefore, it has neutral character.

Page 100 of 135


Note: The pH of water depends on its source. For example:
i) The pH of distilled water is 7 and it is neutral.
ii) The pH of chlorinated water is less than 7 and it is acidic.
iii) The pH of hard water is more than 7. Therefore, it is basic

Precautions: i) Use separate strips for testing different samples.


ii) Keep the pH strips away from chemical fumes.

Increasingly Increasingly
Acidic basic
Acidic Neutral Basic

0 1 2 3 4 5 6 7 8 9 10 11 12 13 14

Page 101 of 135


PRACTICAL BASED QUESTIONS:

1. A solution ‘X’ gives orange colour when a drop of universal indicator is added to it. On the
other hand, another solution ’Y’ gives bluish-green colour with universal indicator. What type
of solutions are ‘X’ and ‘Y’ and are their pH values?
2. On adding a few drops of universal indicator to three unknown colourless solutions P, Q, and
R taken separately in three test tubes, a student observed the changes in colour as green in P,
red in Q and violet in R.
(a) Write the decreasing order of their pH
(b) In which test tube solution is acidic in nature?
3. A student has 3 samples A, B, and C containing dil HCl, dil NaOH and distilled water
respectively. What colours will be observed when these samples are tested with pH paper and
what are their approximate pH values?
4. A student dips pH papers in solutions A and B and observes that the pH paper turns blue and
orange respectively. What could be his inference?
5. Two students were given 3 colourless liquids each of A (water), B (lemon juice) and C (soap
solution). After testing these liquids with pH paper, the following sequence of colour change
on pH paper was reported.
Student 1 – Green, Red, Blue

Student 2 – Blue, Green, Red

Which student reported the correct sequence. Explain.

6. Five solutions P, Q, R, S and T when tested with universal indicator showed Ph of 13, 8, 1, 5
respectively.
A) Which solution is strongly alkaline and which is weakly acidic?
B) Arrange the pH in increasing order of H+ ion concentration.
7. A student was given 4 unknown colourless samples labelled A, B, C and D and was asked to
test the pH. He observered the pH turned liht green in A, dark red in B, light orange in C, and
dark blue in D. What is the correct sequence of increasing order of the pH value of the
samples?
8. Bottle A contains oxalic acid, bottle B contains sodium carbonate solution. When pH paper is
dipped, what colour is observed in bottle A and B?
9. On putting few drops of unknown solution on pH strip, the colour changes to violet. What is
the nature of the liquid likely to be?
10. The pH of NaOH solution is 10.6. On addition of water to this solution, what will happen to it
pH. Explain.

Page 102 of 135


Answers:
1. X is a weak acid and Y is a weak base.
2. (a) Q < P < R
(b) Solution in test tube Q is acidic in nature.

3. A with dil HCl gives Red colour, B with dil NaOH gives blue and distilled water gives green
colour.
4. Solution A is a Base and solution B is an acid.
5. Student 1 reported correct. Water is neutral and give green colour, lemon juice is acidic give
red colour and soap solution is basic which gives blue colour on pH paper.
6. a) Solution P is strongly alkaline and Soution T is weakly acidic.
b) P<Q<S<T<R

7. B<C<A<D
8. Orange and blue
9. Strong base like NaOH
10. PH of solution decreases on dilution as the no. of H+ ions decrease per unit volume.

Page 103 of 135


-
EXPERIMENT NO.4

Objective: To study the interaction of metals (Zn, Fe, Cu and Al) with their salt solutions and arrange
them according to their decreasing reactivity.

Materials required: Zinc granules, iron filings, aluminium foil, zinc sulphate solution, ferrous
sulphate solution, copper sulphate solution, aluminium sulphate solution and test tubes.

Theory:
• More reactive metal can displace the less reactive metal from their salt solution. These
reactions are called displacement reactions.
• Al is more reactive than Zn, Fe and Cu; therefore, it can displace these metals from their salt
solutions.
2Al + 3ZnSO4 Al2(SO4)3 + 3Zn
2Al +3 FeSO4 Al2(SO4)3 + 3Fe
2Al + CuSO4 Al2(SO4)3 + 3Cu
• Zn is more reactive than Fe and Cu; therefore, it can displace these metals from their salt
solutions.
Zn + FeSO4 ZnSO4 + Fe
Zn + CuSO4 ZnSO4 + Cu
• Fe is more reactive than Cu; therefore, it can displace the metal from its salt solution.
Fe + CuSO4 FeSO4 + Cu

Name of the substance Formula Original colour


zinc sulphate ZnSO4 Colourless
ferrous sulphate FeSO4 Light green
copper sulphate CuSO4 Blue
aluminiumsulphate Al2(SO4)3 Colourless

Procedure and observation table:

S.No
Experiment Observation Inference
1 Take 4 test tubes and add
Al foil Colourless solution, Zn Al is more reactive
a) To ZnSO4 solution. metal gets deposited on than Zn.
aluminium.

Pale green solution Al is more reactive


b) To FeSO4 solution. changes to colourless than Fe.
solution and grey

Page 104 of 135


coloured iron metal get
deposited. Al is more reactive
than Cu.
c) To CuSO4 solution. Blue solution changes to
colourless solution and
reddish brown copper
metal gets deposited. There is equilibrium
d) To aluminiumsulphate There is no change between Al3+ and Al.
solution

S.No
Experiment Observation Inference
2 Take 4 test tubes and
add Zn granules

a) To FeSO4 solution. Pale green solution changes Zn is more reactive than


to colourless solution and Fe.
iron metal gets deposited.

b) To CuSO4 solution. Blue solution changes to Zn is more reactive than


colourless solution and Cu.
reddish brown copper metal
gets deposited on zinc.

c) To Al2(SO4)3 There is no change Zn is less reactive than


solution. Al.

d) To ZnSO4 solution. There is no change There is equilibrium


between Zn2+ and Zn.
3 Take 4 test tubes and
add iron filings
a) To CuSO4 solution. Blue solution changes to Fe is more reactive than
green solution and reddish Cu.
brown copper metal gets
deposited.

b) To Al2(SO4)3 There is no change Fe is less reactive than


solution. Al.
Fe is less reactive than
c) To ZnSO4 solution. There is no change Zn.
There is equilibrium
d) To FeSO4 solution. There is no change between Fe2+ and Fe.

Page 105 of 135


4 Take 4 test tubes and
add copper turnings

a) To ZnSO4 solution. There is no change Cu is less reactive than


Zn.

b) To FeSO4 solution. There is no change Cu is less reactive than


Fe

c) To Al2(SO4)3 There is no change Cu is less reactive than


solution. Al
There is equilibrium
d)To CuSO4 solution. There is no change between Cu2+ and Cu.

Result: The reactivity order of the metals is


Al> Zn> Fe> Cu

Page 106 of 135


PRACTICAL BASED QUESTIONS:

1. Which of the following pairs will undergo displacement reaction? Give reason.
(a) FeSO4 and Cu
(b) FeSO4 and Al
2. Why does Zn react with Sulphuric acid to give hydrogen gas, but Cu does not? Give reason.
3. When a strip of Zn metal is put in Copper sulphate solution, the blue colour of copper
sulphate fades gradually. Why?
4. In an experiment each of the metals Al, Zn, Cu and Fe are reacted with ZnSO4, CuSO4,
Al2(SO4)3 and FeSO4 solutions. The results are tabulated as follows where ‘√’ means that the
reaction occurs and ‘x’ means no reaction occurs. Based on the result, what would be the
order of reactivity of the metals? Explain.

Metal ZnSO4 CuSO4 Al2(SO4)3 FeSO4

Zn X √ X √

Cu X X X X

Al √ √ X √

Fe X √ X X

5. Iron fillings are added to different test tubes each containing aqueous solution of
(a) ZnSO4 (b) CuSO4 (c) FeSO4 and (d) Al2(SO4)3.
In which test tube reaction will not occur and why?
6. Two beakers A and B contain iron sulphate solution. In beaker A, small pieces of copper is
added. In beaker B, small pieces of zinc is added. It was found that grey deposit is formed on
zinc in beaker B but no change on copper in beaker A. What is the conclusion you can derive
from these observations?

7. A student took 4 test tubes A, B, C and D containing aluminium sulphate, copper sulphate,
ferrous sulphate and zinc sulphate solutions respectively. He placed an iron strip in each of
them. What will be the observation in the test tubes?

8. A piece of granulated zinc was dropped into copper sulphate solution. After sometime the
colour of the solution the colour of the solution changed. Explain the observation.
Page 107 of 135
9. Based on the sequence of the reactions, identify the most and reactive elements
A + BX → AX + + B

C + AY → CY + A

10. On placing a piece of aluminium metal in the solution of mercuric chloride, it acquires a
shining silvery surface but when it is placed in a solution of magnesium sulphate, no change is
observed. Why?

Answers:
1. Al is more reactive than Fe hence displacement reaction takes place.
2. Zn is more reactive and placed above hydrogen in the reactivity series hence H2 gas is
released. Cu is less reactive and hence cannot displace H2.
3. Zn displaces Cu from CuSO4 and forms ZnSO4 which is colourless.
4. Al, Zn, Fe, Cu
5. Iron filings will not react with ZnSO4, FeSO4 and Al2(SO4)3
6. Zn is most reactive metal followed by iron and then copper.
7. In test tube B copper is deposited on iron strip and in all other test tubes, no reaction is
observed.
8. Blue colour copper sulphate becomes colourless as zinc replaces copper from copper sulphate
solution to form zinc sulphate.
9. C is most reactive and B is least reactive.
10. Aluminium is more reactive than mercury and it displaces mercury from mercuric chloride
solution and mercury deposits on aluminium solution giving it silvery shiny surface.
Aluminium is less reactive than magnesium. Hence cannot displace magnesium from
magnesium sulphate. Hence no change is observed.

Page 108 of 135


EXPERIMENT NO.5

Objective: To study the physical and chemical properties of acetic acid (ethanoic acid).

Materials required: Acetic acid, test tubes, litmus paper (blue and red), sodium bicarbonate and
distilled water.

Theory:

Acetic acid is an organic acid with the formula CH3COOH.Its functional group is –COOH group.
It is a weak monocarboxylic acid since it has only one carboxylic group. This acid ionizes in polar
media to give H+ ions which are responsible for its acidic nature.

CH3COOH(aq)⇌CH3COO- (aq) + H+(aq)


Pure acetic acid is also known as glacial acetic acid because it freezes to form ice like crystals.

Acetic acid has vinegar like smell.


Acetic acid turns blue litmus paper red.

Acetic acid produces effervescence with sodium bicarbonate liberating carbon dioxide gas.
CH3COOH(aq) + NaHCO3(s) CH3COONa(aq)+H2O(l)+ CO2(g)

Carbon dioxide turns lime water (solution of calcium hydroxide) milky due to the formation of
insoluble calcium carbonate.
Ca(OH)2(aq) + CO2(g) CaCO3(s) + H2O(l)

Page 109 of 135


Procedure and Observation table:
S.No Experiment Observation Inference
Physical Properties:
a) Take a small amount of acetic Colourless liquid Acetic acid is a
acid in a test tube and observe its having pungent smell colourless liquid having
colour and odour. irritating pungent smell.

b) Dissolve acetic acid in the Colourless Acetic acid is highly


distilled water. homogeneous soluble in water.
solution obtained.

c) Take a small amount of acetic Blue litmus turns red. Acetic acid is acidic in
acid in a test tube and dip a strip of nature.
blue litmus paper in the solution.

d) Take a small amount of acetic No change in the Acetic acid is acidic in


acid in a test tube and dip a strip of colour of red litmus nature.
2 red litmus paper in the solution paper.

Chemical properties:
a) Take sodium bicarbonate in a dry Brisk effervescence is When sodium
test tube and add slowly few drops evolved. bicarbonate reacts with
of acetic acid. acetic acid it produces
CO2 gas.

Precautions:
1) Do not inhale the vapours of acetic acid directly.
2) Carbon dioxide should be passed through freshly prepared limewater only, for a short duration
3) Acetic acid should be handled with care.

Page 110 of 135


PRACTICAL BASED QUESTIONS:
Q1) Give the IUPAC name, molecular formula and structural formula of Acetic acid
Q2) Write two tests you would perform to detect, whether the given colourless liquid is acetic acid or
not.
Q3) What happens when acetic acid is added in a solution of Na2CO3/NaHCO3 in a test tube? Write
the chemical equation for the same.
Q4) what is the freezing temperature of ethanoic acid? Write the name & formula of the ester formed
when ethanoic acid reacts with ethanol
Q5) Name the functional group and alkyl group present in acetic acid.
Q6) Give a test for carbon di oxide also write the chemical equation for the test. Write the formula of
freshly prepared lime water.
Q7) Give four precaution to be taken while performing this experiment
Q8) How can you prepare fresh lime water? What is it used for?
Q9) What happens when acetic acid solution is added to Na2CO3 in a test tube? Write the
equation for detecting the gas evolved.
Q10) Why should we not pass excess of CO2 through lime water? What is vinegar?

Answers
Q1) Ethanoic acid, CH3COOH,

Q2 Test 1:- If we put a drop of the given colourless liquid on blue litmus paper , it will change to red.
Test 2:- If we smell the given liquid it will smell like that of vinegar.

Q3) Brisk effervescence of gas is evolved. Sodium acetate, water and CO2 is formed.

Q4) 16.60C or 289.6K, Ethyl Ethanoate, CH3COOC2H5

Q5) Carboxylic group and methyl group

Q6) Ca (OH)2

Page 111 of 135


7) Acetic acid should be handled with care.
Only small amount of chemicals should be used.
Fresly prepared lime water should be used.

Page 112 of 135


Experiment 6

Aim:
To study saponification reaction for preparation of soap.
Theory
Soaps are sodium or potassium salt of higher fatty acids such as Oleic acid, stearic acid, palmitic acid
etc.
Glycerides: they are esters of glycerol, an alcohol containing three hydroxyl groups and fatty acids.
Glycerides are present in fats or oils of plants and vegetables.

Saponification: it is process in which esters are split with the help of an alkali. Esters are reacted
with sodium hydroxide salt to form sodium salt of acid and alcohol.

Materials Required:
Sodium hydroxide solution, plant oil (use castor oil + oleic acid) and common salt.
Apparatus: Beakers, glass rod, tripod stand, Bunsen burner, wire gauze and pair of tongs.
Procedure:
1. Take a beaker and add 20ml. of castor oil into it. Heat the oil by constant stirring.
2. Take 20ml. of concentrated sodium hydroxide solution in another beaker.

Page 113 of 135


3. Add the sodium hydroxide solution into the beaker containing hot castor oil. Heat the mixture
slowly to boil for about 10 mins.
4. Now add 5g. of sodium chloride into this mixture with constant stirring and then allow to cool it.
Addition of common salt decreases the solubility of soap. The soap molecules gets separated from the
solution and floats on the surface. This is called salting out of soap.
5. On cooling the beaker, a crust is formed on the surface of the liquid. This crust is called soap.
6. The castor oil will produce a soap molecule named sodium oleate (C17H33COONa).
Conclusion:
The reaction mixture of (oil) carboxylic acid with sodium or potassium hydroxide produces a soap
molecule.
Precautions:
1. Be careful while handling concentrated sodium hydroxide as it is corrosive in nature.
2. Do not overheat the beaker while heating procedures.
3. Add very small amount of common salt for salting the soap.

Page 114 of 135


PRACTICAL BASED QUESTIONS:

Q1) What are soaps? Name one by product of soap formation.


Q2) Name the part of soap molecule that attracts water. Give one advantage of soap over detergent.
Q3) What is the nature of the soap? What happens when soap is added to hard water?
Q4) What are detergents? Give one advantage of detergent over soap.
Q5) What is saponification? Why is it advised to add common salt while preparing soap?
6) Mention the essential material (chemicals) to prepare soap in the laboratory. Describe in brief the
test of determining the nature (acidic/alkaline) of the reaction mixture of saponification reaction.
Q7) What is the chemical reaction involved in the manufacture of soap? Can you suggest a method
to separate glycerine from the reaction mixture?
Q8) What are esters? What are the commonly used esters?
Q9) Give 4 precautions to be taken while preparing soap in labarotary.
Q10) What does the hydrophilic and hydrophobic parts of soap means to you? Give two
disadvantages of soap.

Answers
1) Soaps are the potassium or sodium salts of fatty acids. Glycerol
2) Hydrophilic part. Soaps are bio degradable
3) Soap is basic in nature. It turns red litmus blue. Due to the formation of scum the cleansing
property of water is reduced
4) Detergents are sodium salts of long chain benzene sulphonic acid. Detergents can be used in
hard water as well
5) Saponification is hydrolysis of ester under basic condition to form salt of carboxylic acid and
alcohol. Common salt decreases the solubility of soap

6) A 6) Raw Materials Required For preparation of soap in laboratory are :


Vegetable oil , Sodium Hydroxide and Commom salt

Test to determine the nature of reaction mixture: When a red litmus paper is dipped in the
reaction mixture, the paper changes its colour to blue. Hence, the reaction mixture of the
saponification reaction is basic in nature.

Page 115 of 135


7) The chemical reaction involved in the manufacture of soap is as follows:

Glycerine is separated from the reaction mixture by the addition of salt. Solubility of soap in the
reaction mixture decreases, soap curdles and floats to the top. It can then be skimmed off from the
reaction mixture which would then contain glycerine.
8) Esters are carbon compounds with general formula RCOOR’ where R & R’ are alkyl groups.
Fats and oils are commonly used esters.

9)
Q10)

-------------------------------------------------------------------------------------

Page 116 of 135


Experiment 7
Aim:
To study the comparative cleaning capacity of a soap in soft and hard water.
Theory
Soap: it is the sodium or potassium salt of long chained carboxylic acids.
Soap dissolving in water: the ionic end of soap dissolves in water while the carbon chain does not
dissolve in water but can dissolve in oil.
Soft water: the water with no salt in it.
Hard water: the water with dissolved salt in it. It maybe calcium or magnesium salts. Soap after
dissolving in soft water shows the cleansing property by forming foam, but on dissolving in hard
water it forms white scum due to the reaction of soap with the calcium and magnesium salts. The
charged ions off soap combine with these salts already present in water to form the scum.
Cleansing action of Soap:
1. Soaps and detergents have a large hydrocarbon tail (hydrophobic) and a negatively charged head
(hydrophilic)
2. When a soap or detergent is dissolved in water, the molecules gather together as cluster called
micelles. The tail stick towards inwards and head towards outwards as water being polar in nature.
3. When water is agitated, the oily dirt tends to lift off from the dirty surface and dissociate into
fragments. Thus the solution contains small globules of oily dirt surrounded by soap molecules.
4. The oily dirt as soap micelles is removed along with the water used to wash dirty clothes.

Page 117 of 135


Materials required:
Two test tubes, test tube stand, samples of hard and soft water, soap solution and cooking oil.
Procedure (part A):
1. Take 10ml of distilled water (soft water) in a test tube. Label it as ‘A’.
2. Take 10ml of hard water (water from hand pump, underground water) in another test tube.
Label it as ‘B’.
3. In both the test tubes add few drops of soap solution.
4. Shake the test tubes ‘A’ and ‘B’ vigorously for an equal period of time. Keep them in the test
tube stand and record your observations.
Observations:
1. In test tube A, soap formed lather or foam.
2. In test tube B, white precipitate was formed with no lather or foam.
Conclusion:
Soaps are effective cleaner only in soft water because the soap molecules separate in soft water. But
in the case of hard water, the soap molecules do not remain as soap molecules but the ionic end of
soap reacts with the salts present in hard water to form curdy white precipitate called scum.
Note:
Page 118 of 135
If hard water is not available prepare some hard water by dissolving hydrogen
carbonate/sulphate/chlorides of calcium or magnesium in water.
Procedure (Part B):
1. Take 10ml of distilled water/soft water and add a drop of cooking oil in it. Label this test tube as
‘A’.
2. Take 10ml. of hard water and add a drop of cooking oil in it. Label this test tube as ‘B’.
3. Now add a few drops of soap solution in both the test tubes ‘A’ and ‘B’.
4. Shake both the test tubes vigorously for the same period of time.
5. Keep them on the test tube stand and record your observations.
Observations:
1. The test tube ‘A’ with soft water showed the oil emulsified due to soap solution.
2. The test tube ‘B’ showed no emulsification due to soap solution.
Conclusion:
The formation of emulsion of oil in soft water by soap shows that soap is not very effective in
cleaning with hard water. Soaps are more effective cleaners in soft water than in hard water.

PRACTICAL BASED QUESTIONS

1. When boilers of water are used for very long time, then white layers get deposited on inside of
these boilers. How can these white layers be removed?
When hard water is used in these boilers, then white scumof salts present in the hard water gets
deposited inside the boilers making a white layer. These white layers can be removed by washing the
boilers with dil HCl acid.
2. Why are soaps effective in soft water and not in hard water?
In hard water it forms insoluble scum where as in soft water it does not form any such compound and
hence shows its cleansing property.
3. How will you test in the laboratory, whether the given sample of water is hard or soft. Name two
salts which make the water hard.
When few drops of soap solution are added in the given sample of water, if lather is formed with soap
then water is soft water. If lather is not formed, then water is hard. Calcium chloride and magnesium
sulphate salts present in water make the water hard

Page 119 of 135


4. Name two sources of soft and hard water.
Soft water-Rain water, boiled water
Hard water—Sea water, ground water
5. What do you understand by temporary and permanent hardness of water?
Temporary hardness is caused due to the presence of calcium and magnesium bicarbonates in water
and can be removed by boiling or by adding Na2C03 to hard water.
Permanent hardness is caused due to the presence of chlorides and sulphates of calcium and
magnesium and can be removed by using an ion exchanger
6. write any two disadvantages of detergents over soap?
1.The major disadvantage of detergents is that they are non-biodegradable. 2) They cause soil
pollution and water pollution.
7. Advantages of synthetic detergents over soaps
a ) detergent work well even with hard water but soap do not.
b ) detergent may be used in saline or acidic water.
c) detergent are more easily soluble in water than soaps.
d) detergent can be used of washing woolen garments whereas soaps cannot be used

8) What is soft water? Which ions are present in soft water?


The water which has low mineral content and produces lather on mixing soap to it is called soft
water..
Only sodium ion is present in soft water
9) Can hard water be softened? How?
Yes.
By boiling or by chemical treatment.

10 Can you explain why sodium salts in water do not make it hard water?

Soaps are sodium salts of fatty acids. On adding sodium salts to water containing soap, no scum is
formed as can be understood by the following equation.

Page 120 of 135


PERIODIC TEST – I (2019-2020)
Subject: SCIENCE Max. Marks: 30
Grade: X Time: 1hr10min
Name: Section: Roll No:
General Instructions:
• This question paper consists of 2 printed pages.
• All answers to be written in answer sheet provided. Physics, Chemistry and Biology to be
answered on separate answer sheets

CHEMISTRY (10)
1. Identify the substance that is (a) oxidized and (b) reduced in the reaction. (1)
CuO (s) + Zn (s) → Cu (s) + ZnO (s)
2. Dry ammonia gas has no action on litmus paper, but a solution of ammonia in water turn red (1)
litmus paper to blue. Why is it so?
3. A shiny brown coloured element X on heating in air becomes black in colour.
a) Name the element X and the black coloured compound formed. (2)
b) Oil and fat containing food items are flushed with nitrogen. Why?

2g of ferrous sulphate crystals are heated in a dry boiling tube. (3)


4. a) List any two observations.
b) Name the type of chemical reactions taking place.
c) Write the type of chemical equation for the reaction.
5. Explain the action of dilute hydrochloric acid on the following with chemical equations:
a) Magnesium ribbon (3)
b) Sodium hydroxide
c) Marble pieces

Page 121 of 135


ANSWER KEY

PERIODIC TEST – I (2019-2020)


ANSWER KEY
Subject: SCIENCE Max. Marks: 30
Grade: X Time: 1hr10min
CHEMISTRY (10)
1 CuO (s) + Zn (s) → Cu (s) + ZnO (s) ½+½
. (a) Substance oxidized - Zn

(b) Substance reduced --CuO

2 Ammonia dissolves in water, forms ammonium hydroxide which is base and turns red litmus (1)
. blue.
But, Dry Ammonia gas does not release OH- ions in the absence of water
3 a) i) The element X - Cu
. ii) The black colored compound - CuO (1+1)
b) To prevent food items containing oil and fat from being oxidized and turned to rancid,
they are flushed with unreactive gas like nitrogen.

a) i) Green colour of ferrous sulphate disappears and reddish brown Fe2O3 is formed. (1+1+1
4 ii) Smell of burning Sulphur. )
. b). Decomposition reaction
c) 2FeSO4 (s) →. Fe2O3 (s) + SO2 (g) + SO3 (g)
5 a) Hydrogen gas will be formed.
. Mg (s) + 2HCl (aq) → MgCl2 (aq) +H2 (g) (1+1+1
b) Sodium Chloride and water will be formed. )
NaOH(aq) + HCl (aq) → NaCl (aq)+ H2O (l)
c) Brisk effervescence due to CO2 gas.
CaCO3 (s) + 2 HCl(aq) →CaCl2 + H2O(l) + CO2 (g)

Page 122 of 135


FIRST TERM EXAM (2019-20)
Subject: CHEMISTRY Max. Marks: 26
Grade: 10 Time:
Name: Section: Roll No:
General Instructions:
• This question paper consists of 2 printed pages.
• All answers to be written in the answer sheet provided.

I. SECTION A 1m
Multiple Choice Questions:
4. Which of the following is the correct statement?
a More the concentration of H+ ions in the b. Lesser the concentration of H+ ions in the
. solution, lower the pH value. solution, lower the pH value.
-
c Lesser the concentration of OH ions in the d. More the concentration of OH- ions in the
. solution, higher the pH value. solution, lower the pH value.
5. Which one of the following elements is a solid non-metal?
a Mercury b. Sodium
.
c Iodine d. Bromine
.
6. Among the following metals, the poorest conductor of heat is:
a Lead b. Iron
.
c Aluminium d. Tin
.
7. In the equation C4H10 + X O2 → 4CO2 + 5 H2O, the value of X is:
a 4 b. 5
.
c 13 d. 10
.

VSA 1m
15. Astha has a good collection of coins. One day she observed a green coating on one of the
Page 123 of 135
copper coins. Which phenomenon is responsible for this coating? Write the chemical
name of the green coating.
OR
During electrolysis of water two types of gases are released. How can we identify oxygen
among them?

16. Give one example of:


a) Liquid Non-metal
b) Metal which can be cut easily with a knife
17. Bells are made up of metals and not wood. Which property of metal can be explained by
this?

II. SECTION B 3m
24.
Coal is burnt in air when used as fuel in industry or in thermal power stations.
a) Write the balanced chemical equation with state symbols for burning of coal.
b) Write 2 types of reaction this can be classified.
OR
Lead nitrate is strongly heated.
a) What type of chemical reaction takes place?
b) Write a balanced chemical equation with colour change and state symbols.
25. How is bleaching powder produced? Write a balanced chemical equation for the
formation of Bleaching powder and two industrial application of Bleaching powder.
26. Explain an activity with a proper format (Aim, procedure, observation, chemical reaction
and Inference) to show that Exothermic and Displacement reaction takes place when zinc
granules is made to react with dilute HCl.

II. SECTION C 5m

33. (i) a) Name the hardest chemical substance in the body, of which tooth enamel is
made.
b) How does tooth enamel get corroded?
c) How does toothpaste protect tooth decay?
(ii) Indigestion or fasting can cause irritation or burning sensation and pain in the
stomach. What could be the reason for this. What remedy can you suggest this?
OR
a) What is Chlor - alkali process?
b) Write the chemical reaction taking place in the form of a balanced chemical
equation.
c) Name the gases given off at the anode and the cathode respectively.
d) Write one use each of any two products produced in this process.
34. (i) A reddish-brown metal X when heated in presence of oxygen forms a black compound
Y. When Y is heated with Hydrogen gas it gives back X.
a) Identify X and Y.

Page 124 of 135


b) Write a balanced chemical equation for heating of X in presence of oxygen.
c) Write a balanced chemical equation for the reaction between Y and Hydrogen gas.
(ii) Identify the substance oxidized and reduced in the reaction between Y and Hydrogen
gas.

ANSWER KEYS FIRST TERM EXAM (2019-20)


Subject: CHEMISTRY Max. Marks:26
Grade: 10 Time:
Name: Section: Roll No:
General Instructions:
• This question paper consists of ? printed pages.
• All answers to be written in the answer sheet provided.
I SECTION A
.
Multiple Choice Questions: 1x4
4 a
5 c
6 a
7 c

VSA
15. Corrosion, Bacic Copper Carbonate - CuCO3.Ca(OH)2 ½+½
OR
The gas produced in lesser quantity is oxygen. It can be tested by bringing a burning 1
match stick to the mouth of the test tube, it continues to burn brightly as oxygen is a
supporter of combustion.

16. a) Bromine ½ +½
b) Lithium or Sodium or Potassium

17. The property of Sonorous which is the ringing sound produced when a metal is struck 1

II. SECTION B
24. a) C (s) + O2 (g) → CO2 (g) + Heat 1+2
Page 125 of 135
b) Combination reaction and Exothermic reaction
OR
a) Decomposition reaction and Endothermic reaction 1+2
b) 2Pb(NO3)2 (s) → 2Pb2O (s) + 4NO2 (g) + O2 (g)

25 Bleaching powder is produced by the action of Chlorine on slaked lime (Calcium 1


Hydroxide).
1
Ca(OH)2 + Cl2 → CaOCl2 + H2O
Bleaching cotton and linen in textile industry or bleaching wood pulp in paper industry ½+½
Oxidising agent in chemical industry

26 NCERT book Pg 2 Activity 1.3 3

II. SECTION C
33 (i) a. Calcium Phosphate 1
b. When pH of the mouth is below 5.5 tooth enamel get corroded 1
c. Tooth paste is basic in nature, it neutralizes excess acid and prevents tooth 1
decay
(ii) Production of HCl in stomach cause pain. Antacid is treated which is basic in 2
nature and neutralizes the acid.
OR
a) When Brine undergoes electrolysis it decomposes to produce 3 important 1
products – NaOH, Cl2 and H2. Hence it is called Chlor-Alkali process.
b) 2NaCl (aq) + 2H2O(l) → 2NaOH(aq) + Cl2(g) + H2 (g) 1
½+½
c) Anode – Cl2 Cathode – H2 1+1
d) One use of any 2 products - H2 or Cl2 or NaOH

34 (i) a. X = Cu, Y = CuO ½+½


b. 2Cu + O2 → 2CuO 1
c. CuO + H2 → Cu + H2O 1
1+1
(ii) CuO is reduced to Cu and H2 is oxidized to H2O

Page 126 of 135


PERIODIC TEST – II (2019-2020)
Subject: SCIENCE Max. Marks: 30
Grade: X Time: 1hr10min
Name: Section: Roll No:
General Instructions:
• This question paper consists of 2 printed pages.
• All answers to be written in answer sheet provided. Physics, Chemistry and Biology to be answered
on separate answer sheets

CHEMISTRY (10)
1. Write the name and structure of next homologue of propanol. (1)
2. What are the two properties of carbon which lead to the huge number of carbon compounds we (1)
see around us?

3. Name the functional group present in each of following compounds:


a) HCHO (2)
b) C2H5COOH

a) Why are most carbon compounds poor conductors of electricity? (3)


4. b) Write the molecular formula and electron dot structure of ethene.
c) Give reason: Diamond has high melting point, even though it is a covalent compound.
5. a) What is meant by isomers?
b) Draw and name the isomers of butane. (3)
c) Explain why we cannot have isomers of first 3 members of alkane series.

Page 127 of 135


ANSWER KEY
PERIODIC TEST – II (2019-2020)
Subject: SCIENCE Max. Marks: 10
Grade: X Time: 1hr10min
Name: Section: Roll No:
General Instructions:
• This question paper consists of 2 printed pages.
• All answers to be written in answer sheet provided. Physics, Chemistry and Biology to be answered on
separate answer sheets

CHEMISTRY (10)
1. Butanol, CH3CH2CH2CH2OH. (1)
2. 1. Catenation (1)
2. Tetravalency

3. :
a) HCHO -ALDEHYDE (-CHO) (1+1)
b) C2H5COOH - CARBOXYLIC ACID ( -COOH)

a) Due to covalent bond. (ions are absent) (1+1+1)


4. b) molecular formula C2H4
electron dot structure of ethene.
c) : Diamond has high melting point, due to strong 3D structure..
5. a) isomers are compounds having same molecular formula but different structures.
b) isomers of butane. -n- butane and isobutane (1+1+1)
Sructures Pg.65
c) isomers of first 3 members of alkane series are not possible because branching of carbon
atoms is not possible.

Page 128 of 135


PRE-BOARD-1 (2019-2020)
Subject: Chemistry Max. Marks:25
Grade: 10 Time:
Name: Section: Roll No:
General Instructions:
• This question paper consists of 2 printed pages.
• All answers to be written in the answer sheet provided.
SECTION-A (1 mark)
1. What type of chemical reaction take place when magnesium wire is burnt in the air? 1
2. What is the nature of non-metal oxides when dissolved in water? 1
10 Which of the following gives CO2 on heating? 1
.
a) Slaked lime
b) Quick lime
c) Limestone
d) Soda ash

11 Which of the following is amphoteric in nature? 1


.
a) Na2O
b) MgO
c) CaO
d) Al2O3

Page 129 of 135


12 Heating a sulphide ore is called- 1
.
a) Smelting
b) Calcination
c) Refining
d) Roasting
OR
In thermite process, the reducing agent is-
a) Nickel
b) Zinc
c) Aluminum
d) Sodium

13 Assertion (A): Many carbon compounds are formed due to catenation. 1


Reason (R): Carbon compounds show the property of allotropy.
Select the correct answer to these questions from the codes (i), (ii),(iii) and (iv) as given
below:
a) Both A and R are true, and R is correct explanation of the assertion.
b) Both A and R are true, but R is not the correct explanation of the assertion.
c) A is true but R is false.
d) A is false but R is true.
SECTION-B (3 marks)

15 A reddish- brown vessel developed a green colored solid ‘X’ when left open in air for a 3
. long time. When reacted with dilute H2SO4, it forms a blue colored solution along with
brisk effervescence due to colorless and odorless gas ‘Z’ which turned lime water milky. X
decomposes to form black colored oxide Y of a reddish- brown metal along with gas Z.
Identify X, Y, & Z.

16 a). Which gas is usually liberated when an acid reacts with a metal? 3
.
b). Illustrate with an example.
c). How will you test for the presence of this gas?
OR

A yellow powder X gives a pungent smell if left open in air. It is prepared by passing
chlorine gas over a dry compound Y which also finds use in disinfecting water.

Page 130 of 135


a) Identify X and Y

b) Write the corresponding chemical equation for the reaction of Y with chlorine gas.

17 a) Write the IUPAC name of – 3


.

b) Select an alkene from the following:

CH4, C2H2, C2H4, C3H8

c) Draw the electron dot structure of the next homologue of methanol.

SECTION-C (5marks)

25 An element E combines with oxygen to form an oxide of type E2O, which is a good 5
. conductor of electricity. For safety reasons, the element E is kept under kerosene. Based on
the information provided, answer the following questions:

a) How many electrons are present in the outermost (valence) shell of E?

b) Show the formation of E2O by transfer of electrons.

c) Write the formula of the compound formed when E combines with chlorine.

d) State the reason for the following:

(i) Reactivity of Al decreases if it is dipped in HNO3.

(ii) Carbon cannot reduce the oxides of Na or Mg.

OR

a) Given below are the steps for extraction of copper from its ore. Write the reactions
involved.

(i) Roasting of copper (I) sulphide.

(ii) Reduction of copper (I) oxide with copper (I) sulphide.

(iii) Electrolytic refining

b) Draw a neat and well labeled diagram for electrolytic refining of copper.

Page 131 of 135


26 a) Complete the reactions and write the names of the products formed: 3+2
.
i) CH3COOH + NaOH Heat→

ii) C2H5OH Alkaline KMnO4


iii) CH3COOH +C2H5OH Conc.H2SO4


b) A substance ‘A’ works well with hard water and is an ingredient of shampoos. ‘A’ is not
100% biodegradable and causes water pollution. ‘B’ does not work well with hard water but
is 100% biodegradable and does not cause water pollution. Identify ‘A’ and ‘B’.

Page 132 of 135


Answer key
PRE-BOARD-1 (2019-2020)
Subject: Chemistry Max. Marks:25
Grade: 10 Time:
Name: Section: Roll No:
General Instructions:
• This question paper consists of 2 printed pages.
• All answers to be written in the answer sheet provided.
SECTION-A (1 mark)
1. Combination/oxidation
2. Acidic
10 c) Limestone
.
11 d) Al2O3
.
12 d) Roasting
.
OR
c) Aluminum

13 c) A is true but R is false.

SECTION-B (3 marks)

Page 133 of 135


15 X- Basic copper carbonate; CuCO3.Cu(OH)2
.
Y- CuO
Z- CO2

16 a) Hydrogen gas
.
b) Zn + HCl → ZnCl2 + H2

c) On bringing a burning candle or matchstick to the mouth of test tube, H2 gas


burns with a pop sound.

OR

a) X – CaOCl2 and Y- Ca(OH)2

b)

17 a) IUPAC name – 2-Methylpropane


.
b) Alkene – C2H4

c) Electron dot structure of ethanol.

SECTION-C (5marks)

25 a) 1 electron 1
. 1
b) Show by electron transfer, the formation of E2O.
1
c) ECl
d) Reason-
i) Since nitric acid is an oxidizing agent, it reacts with aluminium and forms a layer 1
of aluminium oxide (Al2O3) over metal surface which further reduces the reactivity
of metal.
Page 134 of 135
ii) The two metals are placed above carbon in the reactivity series and have a high 1
affinity for oxygen than carbon and thus cannot be reduced by it.
OR

At cathode: Cu+2 (aq) + 2e- →Cu(s) ½+1/2

At anode: Cu(s) →Cu+2 (aq) + 2e-


2

Complete labeled diagram-

26 a) Complete the reactions-


. ½+1/2
i) CH3COOH + NaOH Heat→ CH3COONa +H2O
Sodium ethanoate ½+1/2
ii) C2H5OH + 2[O] Alkaline KMnO
4→ CH3COOH + H2O
½+1/2
Ethanoic acid
iii) CH3COOH + C2H5OH Conc.H2SO4
→ CH3COOC2H5 +H2O 1+1
Ethyl ethanoate

b) A is detergent and B is soap

Page 135 of 135

You might also like